Sie sind auf Seite 1von 64

Agency​: 

A. Basics  
o Agent: any person who is authorized to act on behalf of another (principal) 
o Agency: Agency is the fiduciary relationship that arises when: (1) one person (a “principal”) manifests 
assent to another person (an “agent”), (2) that the agent shall act on the principal's behalf (3) and subject 
to the principal’s control, (4) and the agent manifests assent or otherwise consents so to act. 
o The law of agency governs the interactions among principals, agents, and the third parties with whom 
agents deal on behalf of principles 
o The authority of an agent is created either by deed, by simple writing, by parol, or by mere employment, 
according to the capacity of the parties, or the nature of the act to be done 
o Agency is a fiduciary relationship based on trust 
▪ Involves cost  
▪ Shares information with agent  
B. Agency Formation  
o Consensual – not contractual  
▪ A contract is not required to form an agency relationship because don't need consideration for 
an agency like in a contract 
▪ Restatement § 1: Agency is a relationship that results from: 
● The manifestation of consent by P to A that A shall act  
o P’s behalf  
o Subject to P’s control 
▪ Consideration not required 
▪ Intent to create agency relationship not required 
o Definition (Restatement 3d): Agency is the fiduciary relationship between principal and agent that arises 
when: 
▪ Principal and agent manifest their assent 
▪ that the agent shall act on the principal's behalf  
▪ and subject to the principal’s control. 
● Control is a question of fact determined based on the totality of circumstances. 
o To have control: (always decisive factor) 
▪ Need authority 
▪ Need way to punish if directions not followed 
▪ Court use totality of facts and circumstances to determine control  
● On exam use the ​Nears​ factors (detailed operations manual, training, funds comingling, 
etc.). 
o HHFI had no ownership interest in or financial control over the hotel and 
provided no funds for operating expenses, managers' salaries, or for any other 
purpose 
o There is no comingling of funds or sharing in profits or losses between HHFI 
and the Mount Pleasant hotel 
o HHFI was not involved in any decision with respect to the terms and conditions 
of employment for workers at the hotel 
o HHFI did not have authority to hire or fire workers at the hotel 
o HHFRI did not pay any compensation to workers at the hotel, and paid no 
employment taxes on its behalf 
o While she worked at the Mount Pleasant hotel, Nears did not report to anybody 
at HHFI 

1
o Nears received her paycheck from ETEX while employed by Holiday Inn, the 
limited partnership that owns the hotel 
C. ​Fiduciary Duties (Inward-looking consequences) 
o Inward​ ​looking consequences​: Relate to the relationship between the principal and the agent and are 
largely governed by the contracts between the parties and by the law of fiduciary duties 
o A fiduciary relationship is one that obligates one person to act on behalf and for the benefit of another 
person as a matter of duty.  
o These duties are mandatory – can’t contract around them 
▪ Need these about contractual duties because don’t always have a contract in agency 
1. Principal’s​ ​Duties to Agent 
▪ Performance of contract obligations 
▪ Indemnification 
● indemnify the agent for any payments or expenses properly incurred in the 
course of the agency. 
▪ Good faith and fair dealing 
● somewhat expansive duty and can encompass everything from providing the 
agent with the necessary information for the agent to perform his or her duties 
to the possible risks the agent may be required to take in the course of the 
relationship.  
● requires the principal to cooperate with the agent in the performance of the 
agent’s duties and avoid unreasonably interfering with the relationship. 
2. Agent’s Duties to Principal 
▪ Performance of contract obligations 
▪ Duty of care, competence, diligence 
● would ordinarily be exercised by an agent in similar circumstances. Although this 
is an objective standard, agents who have particular knowledge or skills are 
expected to use them and will be held to a standard of care that would be 
observed by an agent with those special skills.  
▪ Duty of obedience 
● obligation to obey the lawful and reasonable instructions of the principal in the 
carrying out of the agent’s duties. 
● not​ be obligated to follow instructions that are illegal or unduly burdensome and 
onerous 
● duty to conduct herself reasonably and avoid acts that will harm the principal.  
▪ Duty of good conduct 
▪ Duty of disclosure 
▪ Duty to manage of principal’s property 
▪ Duty of loyalty (fiduciary duty to act loyally for the principal's benefit in all matters connected 
with the agency relationship) ​bigger deal to have ownership interest in an adverse party, 
not that agent did not tell the corporation about that interest 
● The agent’s duty of loyalty is multifaceted and includes, among other obligations, a duty 
not to use or communicate confidential information of the principal for the agent's own 
purposes or those of a third party, a duty not to compete with the principal in any matter 
within the scope of the agency relationship, and a duty not to act as an adverse party to 
the principal in a transaction connected with the agency relationship (Food Lion) 
● Material benefit arising out of position: An agent has a duty not to acquire a material 
benefit from a third party in connection with transactions conducted or other actions 

2
taken on behalf of the principal or otherwise through the agent's use of the agent's 
position. 
● Acting as an adverse party: An agent has a duty not to deal with the principal as or on 
behalf of an adverse party in a transaction connected with the agency relationship. 
● Competition: Throughout the duration of an agency relationship, an agent has a duty to 
refrain from competing with the principal and from acting on behalf of or otherwise 
assisting the principal's competitors. During that time, an agent may act, not otherwise 
wrongful, to prepare for competition following termination of the agency relationship. 
● Use of principal’s property and confidential information: An agent has a duty (1) not to 
use property of the principal for the agent's own purposes or those of a third party; and 
(2) not to use or communicate confidential information of the principal for the agent's 
own purposes or those of a third party. 
● WAIVER: ​can seek the principal’s informed consent to engage in an activity that would 
otherwise be a breach of the duty. Such a waiver will only be effective if the agent 
discloses all material facts and otherwise acts in good faith.  
3. Breach of fiduciary  
▪ This is a separate claim  
▪ Breach of contract is a second claim as well as tort  
4. Non-Compete Agreements 
▪ As a contractual extension of duty of loyalty after the termination of the agency relationship  
● Are enforced if reasonable: 
o The restraint is reasonable if: 
▪ It is no greater than what is required for the protection of the employer 
(actual or threatened harm to employer) 
▪ Does not impose undue hardship on the employee; and 
▪ Is not injurious to the public 
● If the restraint is not reasonable, it is enforced to the extent necessary to protect 
employer’s legitimate interests 
D. ​Principles of Attribution (outward-looking consequences)​ – when is the principal liable for acts of agent? 
o Outward looking consequences​: Related to the relationship among the principal, the agent, and a 
third party and are governed by the various principles of attribution 
o The outward looking consequences come about because principals can be held liable for the tortious 
actions of their agents, and can be required to fulfill contracts into which their agents have entered 
1. Actual Authority  
▪ An agent acts with actual authority when, at the time of taking action that has legal consequences 
for the principal, the agent reasonably believes, in accordance with the principal's manifestations 
to the agent, that the principal wishes the agent so to act 
▪ Actual authority is created by a principal's manifestation to an agent that, as reasonably 
understood by the agent, expresses the principal's assent that the agent act on the principal's 
behalf. 
▪ When an agent acts with authority, their action has legal consequences for the principal 
▪ Actual authority may exist even though there is no written contract 
▪ Principal’s manifestation ​to the agent 
● Manifestation = written or spoken words or other conduct 
▪ Actual express authority 
● Conveyed orally or in writing 
▪ Actual implied authority 
● Power to do things “proper, usual, and necessary” to fulfill the agency 
3
● Castillo​ ​v. Case Farms of Ohio (​ implied authority) 
2. Apparent Authority  
▪ Apparent authority is the power held by an agent or other actor to affect a principal's legal 
relations with third parties when a third party reasonably believes the actor has authority to act 
on behalf of the principal and that belief is traceable to the principal's manifestations 
● The principal's manifestations don’t have to be directed to 3rd party; can be to public 
▪ Apparent authority is created by a person's manifestation that another has authority to act with 
legal consequences for the person who makes the manifestation, when a third party reasonably 
believes the actor to be authorized and the belief is traceable to the manifestation 
▪ A person manifests assent or intention through written or spoken words or other conduct 
▪ One​ ​person may bind another in a transaction with a third person, when the third party 
reasonably believes – based on the manifestations of the purported principals- that the 
actor is authorized to act on behalf of the purported principal 
▪ May be basis for liability in 2 situations: 
i. Persons appear to be agents, even though they don't qualify under the definition 
ii. Where agents act beyond the scope of their actual authority 
▪ Communications from principal need not be directed to third persons before apparent authority 
is created 
▪ The Restatement Third clearly permits the creation of apparent authority when manifestations 
reach the third party through an intermediary or simply by placing a person in a certain position 
▪ The manifestation MUST emanate from the principal and must be received by the third person 
and the scope of the agent's apparent authority 
▪ Courts will consider prior dealings between the parties, customs that apply in the particular 
setting, and the nature of the proposed transaction 
▪ Bethany Pharmacal Co. v. OVC, Inc. 
3. Agency by Estoppel  
▪ Also requires the third party to change position in reliance on the principal 
▪ Estoppel is a non-agency doctrine similar to apparent authority 
▪ Two things distinguish estoppel from apparent authority: 
i. Estoppel requires third party to change position in reliance on principal, whereas a 
principal may be bound under apparent authority even in absence of such detrimental 
reliance 
ii. Estoppel allows third party to hold principal liable but does not give principal any rights 
against third party 
4. Ratification  
▪ Way to affirm contract agent prepared on principal’s behalf 
▪ There must have been some transaction or event involving an unauthorized act 
● At the time of act, the principal must have existed and must have had capacity to 
authorize act 
▪ At time of attempted ratification: 
● Principal must have knowledge of all material facts and 
● Third party didn’t indicate (to principal or agent) intention to withdraw from transaction 
▪ Principal can ratify after the fact 
▪ Adoption – when unauthorized act not done for principal; not their agent 
▪ Novation – substitute one party for another 
5. Inherent Agency Power 
▪ Principal is liable for all acts within the authority usually confided to an agent of that character 

4
▪ An undisclosed principal who entrusts an agent with the management of his business is subject 
to liability to third persons with whom the agent enters into transactions usual in such businesses 
and on the principal's account, although contrary to the directions of the principal 
▪ Inherent Agency Power Definition 
● Inherent agency power is a term used in the restatement of this subject to indicate the 
power of an agent which is derived not from authority, apparent authority or estoppel, 
but solely from the agency relation and exists for the protection of persons harmed by or 
dealing with a servant or other agent 
▪ Rationale: Three situations in which principal may become liable: 
i. General agent does typical type of activity, but in violation of orders 
ii. Agent acts for own purposes in entering transaction which otherwise would be 
authorized 
iii. Agent is authorized to dispose of goods and departs from authorized method of disposal 
6. Undisclosed Principals 
▪ Disclosed principal: ​A principal is disclosed if, when an agent and a third party interact, the 
third party has notice that the agent is acting for a principal and has notice of the principal's 
identity 
▪ Undisclosed principal: ​A principal is undisclosed if, when an agent and a third party interact, 
the third party has no notice​ ​that the agent is acting for a principal 
▪ Unidentified principal​: A principal is unidentified if, when an agent and a third party interact, 
the third party has notice that the agent is acting for a principal but does not have notice of the 
principal's identity 
▪ Liability of Undisclosed Principal 
● An undisclosed principal is subject to liability to a third party who is justifiably induced 
to make a detrimental change in position by an agent acting on principal's behalf and 
without actual authority if the principal, having notice of agent's conduct and that it 
might induce others to change their positions, did not take reasonable steps to notify 
them of the facts 
● An undisclosed principal may not rely on instructions given an agent that qualify or 
reduce the agent's authority to less than the authority a third party would reasonably 
believe the agent to have under the same circumstances if the principal had been 
disclosed 
▪ Agent Liability on the Contract 
● Disclosed principal: No agent liability in most circumstances 
● Two exceptions: 
o Clear intent of all parties that agent be bound 
o Agent made contract but without actual authority 
● Undisclosed principal: Agent treated as though a party to the contract 
o Third party must elect who to sue 
 
 
 
 
 
 
 
 
 
5
scenario for breach  Agent liability  Principal liability  Enforce Contracts against 
of contract  3rd 

Actual Authority  none (unless K says  ALL  Principal 


+ ​disclosed  otherwise) 
principal 

Apparent  can be liable  liable to the third party on any   


Authority +  contracts the third party enters 
disclosed principal  into with the agent based upon 
the apparent authority of the 
agent. 

Actual Authority  party to the contract,  party to the contract and  MAY unless allowing the 
+ ​unidentified  absent an agreement with  consequently liable to the third  unidentified or undisclosed 
principal  the third party that the  party  principal to enforce the 
agent will not be liable.   contract would result in an 
injustice to the third party 

Actual Authority   agent and the third party  liable on the contract.   MAY unless allowing the 
+ undisclosed  are parties to the contract.  unidentified or undisclosed 
principal  liable on the contract.   principal to enforce the 
contract would result in an 
injustice to the third party 
TORT Liability: ​An ​agent is liable to a third party harmed by the agent’s own tortious conduct​, regardless of whether 
the agent acts with any type of authority. A ​principal may be directly liable ​to a third party harmed by an agent’s tortious 
conduct done within the scope of actual authority. ​A principle may be ​vicariously liable under the doctrine of 
respondeat superior​ if the ​agency relationship is a master-servant relationship​, and the ​agent commits the tort ​within the 
scope of employment. 

 
   

6
Termination of authority 
● Death of agent 
● Death of entity 
● death or cessation of principal 
● Restatement - termination only effective once agent has notice 
● Loss of capacity - agent is given notice or adjudication of incompetence 
○ principal may enter into agency agreement that continues after incapacity or vests upon incapacity 
● Time agreed to by the parties 
○ a specified time, or  
○ upon the completion of the principal’s objectives. 
● When the circumstances change so much that the agent should recognize that the principal would no longer 
want to continue the agency.  
● Agencies may typically be revoked by the principal or the agent,  
○ agency powers that are granted as securities and certain proxies may be irrevocable (at least until the 
interest secured is satisfied).  
● Agent breaches his or her fiduciary duties. 
● May be ended by operation of law when statutory provisions so dictate. 
 
Keep in mind that the termination of an agent’s actual authority does not necessarily terminate his or her apparent authority, and the 
principal may need to take additional steps or notify third parties to avoid being bound by unauthorized acts of the agent. 
 
Types of agents: 
● Universal agent 
○ agent who is empowered to engage in any and all activities, of any type, on behalf of the principal. This 
type of agency is quite rare. 
● general agent 
○ authorized to engage in a series of transactions or all transactions of a particular type, sometimes for a 
continuous, ongoing period. 
● Special agent 
○ authorized to engage in a single transaction or a time-limited series of transactions.   

7
Partnership  
A. Advantages v. Disadvantages 
a. Benefits 
▪ Partnerships pass through taxation. Tax is paid by individual partners and in the corporate form 
you get double taxed 
● There are no formalities or filing requirement and you don’t need an agreement 
b. Disadvantages 
▪ Unlimited liability, and liability is joint and several 
B. Formation 
a. Definition  
▪ The association of two or more persons to carry on as co-owners a business for profit forms a 
partnership, whether the persons intend to form a partnership (intention NOT required) 
● Don’t need a contract, just conduct 
● No formalities required  
● General partnerships are not taxed as separate business entities 
▪ Default is partnerships are pass-through taxation  
● Default Rule​: all founding partners share in equally unless expressly state otherwise 
b. Consequences 
▪ One partner may be bound to third parties for acts of another partner 
▪ Partners are personally liable for obligations of the partnership 
▪ If third party can reasonably infer partnership created, then become liable 
c. Has a partnership been formed?​ Courts consider several factors  
▪ Sharing in profits is prima facie evidence of partnership (creates a presumption) 
● If sharing of profits is present, courts examine other aspects of the relationship closely  
● This presumption trumps anything else even if one partner has all the control 
▪ Unless can show a way that will rebut the presumption 
● Payment of debt 
● Independent contractor or other compensation to an employee 
▪ Sharing of losses 
▪ Ownership and control of property and the business  
▪ Does the language of agreement exclude a party from ordinary rights of a partner? 
▪ Actions of a party to the agreement toward third parties (including tax liabilities) 
d. Holmes v. Lerner 
▪ Sharing in profits is not a prerequisite for finding of partnership creation  
C. Management  
a. Management Rights 
▪ Who makes decisions within the partnership? 
▪ Who represents the partnership to third parties? 
▪ RUPA – looks at partnership as entity 
▪ UPA – looks at partnership as a collective 
▪ Each partner has equal rights in the management and conduct of the partnership business  
b. Unanimous consent required 
▪ To authorize amendments to the partnership agreement 
▪ To add new partners 
▪ To authorize acts outside the ordinary course of business 
▪ Vecchitto v. Vecchitto 
● Filing a lawsuit is outside the ordinary course of business  
c. Majority consent required 
8
▪ To authorize acts within the ordinary course of business 
d. Two-person Partnership (p. 45-46) 
▪ Nabisco v. Stroud 
● Dissenting partner liable to third party 
● Fox said weird decision because third party on notice 
▪ Summers v. Dooley 
● Dissenting partner not liable  
▪ Anytime trying to change the status quo 
● Need approval of the majority  
● This requires a vote 
● If second partner doesn’t consent they are not liable  
e. Partner and Partnership Liability 
▪ Will be held liable for each and every act of a partner 
▪ Section 301: (apparent authority in partnership context; any act within ordinary course of 
business binds the partnership) 
● Each partner is an agent of the partnership for purpose of its business.  
● An act of a partner, including execution of an instrument in partnership name, for 
apparently carrying on in ordinary course partnership business or business of the kind 
carried on by partnership binds the partnership, unless partner had no authority to act 
for partnership in the particular matter and person with whom partner was dealing knew 
or had received a notification the partner lacked authority.   
● An act of a partner which is not apparently for carrying on in ordinary course of 
partnership business or business of the kind carried on by partnership binds partnership 
only if act was authorized by other partners. 
▪ Section 308: (makes purported agent liable for transaction) 
● If a person, by words or conduct, purports to be a partner, or consents to being 
represented by another as a partner, in a partnership or with one or more persons not 
partners, the purported partner is liable to a person to whom the representation is made, 
if that person, relying on the representation, enters transaction with the actual or 
purported partnership. 
f. Statement of Partnership Authority  
▪ With respect to all property other than real property, a filed grant of authority is conclusive in 
favor of a person who gives value without knowledge to the contrary. RUPA § 303(d)(1). 
▪ With respect to authority to transfer real property, a statement applicable to property held in the 
name of the partnership and recorded in the office for recording transfers of that real property is 
conclusive in favor of a person who gives value without knowledge to the contrary. RUPA § 
303(d)(2). 
▪ Can also get an affidavit signed by all saying person has authority to act 
D. Fiduciary Duties 
a. Duty of Loyalty  
▪ Traditional Formulation = Meinhard v. Salmon 
● Joint adventurers, like copartners, owe to one another, while the enterprise continues, 
the duty of the finest loyalty. Many forms of conduct permissible in a workaday world 
for those acting at arm’s length, are forbidden to those bound by fiduciary ties. A trustee 
is held to something stricter than the morals of the market place. Not honesty alone, but 
the punctilio of an honor the most sensitive, is then the standard of behavior. As to this 
there has developed a tradition that is unbending and inveterate.” (Know the red 

9
language especially for bar) (the second red sentence is the case takeaway and the 
definition of fiduciary duties) (punctilio = fine point) 
● This is a sweeping holding  
● Salmon had a duty to disclose 
● Salmon had put himself in a position in which thought of self was to be renounced, 
however hard the abnegation  
o Going a little far with the language; self-abnegation = take away of the standard 
▪ RUPA attempts to narrow the broad duties imposed by courts under UPA, providing the duty 
of loyalty is limited to the following: 
● An anti-theft duty that corresponds to the anti-theft duty in UPA §21, except that the 
word formation has been dropped to indicate fiduciary duties arise only after the 
partnership is created 
o A prohibition against self-dealing 
o Duty not to appropriate partnership opportunity 
▪ Line of business test 
● Does the partnership have sufficient experience and ability in the 
field to exploit the opportunity? 
▪ Interest or expectancy test 
● Would opportunity further the established business of the 
partnership? 
o Improper use of confidential information (Gibbs v. BAM) 
▪ A prohibition against competing against the partnership  
● Can’t contract around duty of loyalty but may narrow it  
b. Duty of Care  
▪ UPA contains no provision for duty of care, some courts have implied a duty of care 
▪ RUPA establishes conduct showing gross negligence, recklessness, intentional misconduct, or 
knowing violation of the law would violate the requisite standard of care 
● This is because partners are open to unlimited liability and therefore have an incentive to 
exercise due care and to monitor the behavior of other partners 
● Gross negligence is the standard applied by courts that have implied a duty of care  
▪ While the partners may modify the duty of care by contract, they are not allowed to 
unreasonably reduce the duty of care 
▪ RUPA also adds an obligation of good faith and fair dealing 
c. Good Faith and Fair Dealing 
▪ Not waivable  
▪ Added by RUPA 
E. Financial Attributes  
a. Partnership Accounting  
▪ Capital accounts track each partner’s ownership claim against the partnership  
● Capital accounts represent book value, has nothing to do with value to an outsider  
▪ Capital account is determined by: 
● Contributions 
o Results in a credit to their account  
o Labor not considered to be a capital contribution 
o Normally repaid when a partner withdraws from partnership or partnership 
terminates 
● Share of operational profits or losses 

10
o Profits are normally paid out on periodic basis (often annually) during life of 
partnership 
● Withdrawals (draws) 
o Not automatically entitled to a draw, this must be contracted  
o The partners generally establish the amount of the draws at the beginning of the 
year 
o The draws are earnings that are removed from the partnership  
● Gains or losses upon sale 
o Surplus – proceeds of partnership sale remaining after repaying debts of the 
partnership  
▪ This is distributed to the partners according to the amounts in their 
capital accounts  
▪ After a sale, a surplus results if it is more than the total capital accounts 
(the partnership generated a gain (profit)) 
▪ Any loan a partner makes is not considered to be an ownership interest in the partnership but 
remains a separate obligation of the partnership  
▪ Partners are entitled to repayment of capital contributions or advances made to partnership 
▪ Partnership must reimburse partner for payments made, and indemnify partners for liabilities 
incurred, in the ordinary course of business  
▪ Any partner who makes a payment or advance beyond the amount agreed to be contributed as 
capital is entitled to interest on the amount of the payment or advance  
▪ Advances and indemnification for liabilities incurred often are paid during the course of the 
partnership, but it not, partner have a right to such amounts when the partnership terminates 
and the accounts of the partner are settled  
b. Default Rules for Sharing Profits and Losses 
▪ Partners share equally in profits, regardless of amounts of capital contributions 
● This is independent of original contributions 
● Can contract for this to be any percentage you want 
● But ​Kovacik v. Reed h ​ olds that where one partner contributes money and the other 
partner’s only contribution was skill and labor, neither party is liable to the other for 
contribution of any loss sustained 
▪ Losses are shared in the same proportions as profits 
● Can contract for profits to be shared equally and losses to be proportional to 
contributions 
● If agree on how profits are shared but not on losses, then because of default rule, losses 
will be the same as the agreed upon profits  
c. Order of Repayment of Partnership Liabilities​ (if the partnership assets are insufficient to cover any 
liabilities, partners must contribute to the payment of those labilities)  
i. Amounts owed to creditors of the partnership who are not partners 
ii. Amounts owed to partners other than for capital and profits (partner’s loans) 
iii. Amounts owed to partners for repayment of capital (capital accounts) 
iv. Amounts owed to partners for any remaining profits  
F. Liability to Third Parties 
o Partners may be forced to fulfill the obligations of the partnership to third parties out of their personal 
funds 
o The liability of individual partners for obligations of the partnership may not be altered by agreement 
among the partners because it affects the rights of third parties  
o A partner is bound to third parties for acts of another partner 
11
▪ Liable even for wrongful acts or omissions of any partner acting in ordinary course of business 
▪ A partner cannot escape liability by leaving the partnership and remains responsible for any 
wrongful act or omission (​In re Keck, Mahin & Cate)​  
o Partnerships liability is joint and several 
o Exhaustion requirement: 
▪ Traditionally courts require the plaintiffs go after partnership assets before seeking the personal 
assets of partners 
o Limited Liability Partnerships (​LLP​) 
▪ Requires filing with the state 
▪ Must designate in name that are a limited liability 
▪ Liable for own unlawful acts 
▪ Limited liability partnership means a partnership that has filed a statement of qualification under 
Section 1001 and does not have a similar statement in effect in any other jurisdiction. 
▪ An obligation of a partnership incurred while the partnership is a limited liability partnership, 
whether arising in contract, tort, or otherwise, is solely the obligation of the partnership. A 
partner is not personally liable, directly or indirectly, by way of contribution or otherwise, for 
such an obligation solely by reason of being or so acting as a partner. (All partnership liability is 
joint and several) 
▪ In many states limited to professional entitles (lawyers, accountants, etc.) 
▪ Exhaustion requirement -- courts require plaintiff to go after partnership assets and exhaust 
those before seeking the personal assets of partners 
▪ Note on Limited Partnership (LP) 
1. Governed by ULPA & RULPA 
2. LPs must have at least one general partner (GP), who is responsible for the management 
of the LP, and one limited partner, who is a passive investor. 
o LP law requires that the general partner be subject to unlimited personal liability.   
o Limited partner may be personally liable if he participates in the control of the 
business. 
▪ LPs can register as LLLPs by including a statement to this effect in the certificate of limited 
partnership. 
G. Dissolution  
a. Partnership may be dissolved through: 
i. Dissociation = departure of a partner  
● May trigger the process of winding up  
● If winding up not triggered, the dissociating partner must be bought out  
o If partner is bought out his liability and authority to bind the partnership are 
terminated 
▪ But remember ​In re Keck:​ partner remains liable for wrongs committed 
prior to dissociation 
ii. Express will of all partners 
iii. Event agreed upon in the partnership agreement 
b. Effect of dissociation on rights of partners 
▪ A partner may dissociate at any time 
▪ A key issue is whether the dissociation was rightful or wrongful 
● Resolution often depends on nature of partnership 
● Partnership at will is the default rule 
o Dissociation will always be rightful so no breach and not liable  
● Exceptions to default rule: 
12
o Partnership for a term 
▪ Did the term expire? (even if not, a person can still quit) 
● Yes then dissociation rightful so no breach and not liable 
● No then dissociation is wrongful so breach and partner liable for 
damages 
o Partnership for a particular undertaking 
▪ Was the undertaking complete? (even if not, a person can still quit) 
● Yes, dissociation rightful so no breach and not liable 
● No, dissociation is wrongful so breach and partner liable for 
damages 
▪ Fischer v. Fischer 

 
 
 
   

13
LLC 
A. Advantages 
i. Flexibility 
ii. Tax Treatment under the check-the box-regulation 
● LLC’s can choose if want double taxation or pass-through taxation 
● LLC’s don’t have to meet the factors a corporation does to have pass-through taxation 
iii. Limited Liability  
B. Formation 
▪ Formation process is simple 
● Formed through formal filing of a certification of organization or certificate of formation with 
the state 
o If filing is never made, the default rules don’t adhere 
o After LLC is filed then members enter an operating agreement that sets forth their rights 
and duties 
● Name must reflect company is an LLC 
▪ Operating agreement governs 
● If there is a conflict between certificate of organization and OA 
o OA controls agreements with respect to members, dissociated members, transferees, and 
managers 
o OA controls with respect to third parties who reasonably rely on it 
● In absence of OA, in most states, the default rules on LLC’s are the same as general 
partnerships 
▪ Pre-formation problems: 
● De facto corporation 
o No proper formalities, no de jure corporation (Court may say existed in fact but not law) 
o Promoter may nonetheless be protected from personal liability 
o Many states have moved away from this doctrine because ease of process for formation 
● Corporation by estoppel  
o Recognition of limited liability vis-à-vis third parties who are assuming the promoter is 
acting on behalf of corporation 
o This is using estoppel as a sword instead of a shield 
o Must show third party acted in reliance thinking they were dealing with corp. or LLC 
▪ Stone v. Jetmar 
● To qualify for this doctrine: 
o Must make a bona fide and colorable attempt to perfect an organization 
o Must have statute which allows the corporation to be formed 
o User of the rights claimed to have been conferred by the law 
● Held De facto corporation has been abolished  
o The process is so simple, no one could make a colorable attempt and fail 
● Corporation by estoppel doesn’t apply when the acts forming the basis for an estoppel claim 
are induced by fraud  
C. Management 
▪ The OA can determine management structure  
▪ If not specified in OA, the default rule is all LLC’s are member-managed 
▪ Delaware is very deferential to the OA, so the statute is very generic 
a. Two types of management structure​: 
● Member-Managed 
o RULLCA 
14
▪ Members have equal management rights and decide all ordinary business matters 
by a majority of members 
▪ Extraordinary matters and amendments to the OA require the consent of all 
members 
o DLLCA 
▪ Allocates management rights in proportion to the current percentage or other 
interest of members in the profits of the limited liability company owed by all 
members and decisions made by a majority vote  
● Manager-Managed  
o One partner acts as manager 
o Others are passive investors or members  
o RULLCA 
▪ Managers have exclusive management rights and decide all ordinary business 
matters by a majority of managers 
▪ All members must consent to 
● Sell, lease, exchange, or otherwise dispose of all the company's property 
● Approve a merger, conversion, or domestication under Art. 10 
● Undertake any other act outside the ordinary course of the company's 
activities 
● Amend the operating agreement 
o DLLCA 
▪ Managers have whatever rights are provided in the LLC agreement 
b. Authority to bind LLC 
● If have equal management rights, then can bind the LLC 
● If manager-managed LLC, only managers can bind  
● RULLCA 
o Members have no inherent right to bind LLC 
o Managers hold exclusive power to make decisions and each has equal management rights 
● DLLCA 
o Each member and manager has authority to bind LLC unless provided otherwise in LLC 
agreement 
D. Limited Liability 
a. General Rule​: members of an LLC are not personally liable for any obligation of the LLC simply 
because they are members or managers  
● They are responsible for own acts or omissions and for obligations undertaken by agreement  
● Not personally responsible for acts, omissions, or obligations of other members  
● The limited liability shield ​does not protect one form personal wrongdoing 
b. Liability Shield: 
a) The debts, obligations, or other liabilities of a limited liability company, whether arising in 
contract, tort, or otherwise: (this is the veil) 
1) Are solely the debts, obligations, or other liabilities of the company; and 
2) Do not become the debts, obligations, or other liabilities of a member or manager solely 
by reason of the member acting as a member or manager acting as a manager 
b) The mere failure of a limited liability company to observe any particular formalities relating to 
the exercise of its powers or management of its activities is not a ground for imposing liability 
on the members or managers for the debts, obligations, or other liabilities of the company 
c. Exception: Piercing the LLC veil 
● May pierce the veil when: 
15
i. There is fraud 
▪ Usually not a good theory because must show a subjective intent to prove fraud 
▪ Pleading fraud in federal court requires particularity 
ii. The LLC is a mere instrumentality or alter ego of its owner  
▪ To prevail under the alter-ego theory, a plaintiff need not prove that there was 
actual fraud but must show both: 
● A mingling of the operations of the entity and its owner and 
● An overall element of injustice or unfairness 
▪ A mingling of the operations of the entity and its owner factors: 
● Whether the entity was adequately capitalized for the business 
undertaking; (Does company have money for what deal it is making) 
● Whether the entity was solvent; (did they have enough money; ability to 
pay debts as they become due; look at books to compare assets to 
liabilities and assets must outweigh liabilities to be solvent) 
● Whether dividends were paid/distributions were made 
● Whether entity records were kept; (typical for corporations not LLC) 
● Whether officers and directors/managers functioned properly 
● Whether formalities were observed; (typical for corporations not LLC) 
● Whether the dominant owner siphoned entity funds 
● Whether the entity functioned as a façade for the owner. (did you use 
corporation as extension of yourself) 
● NetJets Aviation, Inc. v. LHC Comm’ns, LLC 
 
E. Fiduciary Duties 
a. Who owes fiduciary duties? 
● Member-Managed – all members owe fiduciary duties (RULLCA) 
o Like a partnership 
● Manager-Managed – only managers owe fiduciary duties  
o Managers owe their duties to the LLC as an entity and other members 
▪ Some statutes impose duties to the individual members of LLC as well 
o Members typically do not owe fiduciary duties to the LLC or other members unless a 
member who act like managers or hold a majority stake  
b. What duty is owed? 
● Most modern LLC statues provide for a duty of loyalty and duty of care, with varying degrees of 
specificity  
● NE – members owe fiduciary duties of loyalty, care, and good faith and fair dealing 
● DLLCA is silent on fiduciary duties  
o Unsettled in DE whether managers and members owed fiduciary duties by default 
o If they exist, they can be modified or eliminated by the LLC agreement 
o Auriga Capital Corp. v. Gatz Properties 
c. What standard is required? 
● Mere negligence is never enough to impose liability on someone 
● Requires gross acts or willful violations  
d. Oppression of the minority 
● Concept from Corporations 
● When a majority member oppresses a minority member, courts may sustain minority owners in 
their claims against an oppressive majority  
F. Dissolution  
16
▪ Depends varying on state 
▪ Exit rights can be negotiated in OA  
 
   

17
Corporations  
A. General Characteristics 
1) Legal personality 
2) Limited liability 
● Owners are protected from being personally liable for the corporation’s obligations 
3) Cost 
● Formalities, accounting, disclosure 
● Must have annual meetings and take minutes 
4) Ownership and control are separate 
● Centralized management 
● Shareholders don’t have much of a voice except for the right to vote, sue, & sell  
5) Double taxation on C-Corps 
● The corporation is taxed on its profit and then if corporation pays out a dividend or distribution, 
the individual who receives this money will also be taxed, so this money taxed twice (seen as a 
negative) 
● But some corporations can be taxed as partnerships (S Corps)  
o This avoids double taxation and will have pass-through taxation 
o To be eligible: (must meet ALL requirements) 
a. Domestic Corporations only 
b. Have only allowable shareholders 
● May be individuals, certain trusts and estates, and may not be 
partnerships, corporations, or non-resident alien shareholders  
c. Have no more than 100 shareholders 
d. Have only one class of stock 
e. Not be an ineligible corporation  
● Certain financial institutions, insurance companies, and domestic 
international sales corporations 
6) Perpetual existence 
● As a default, corporations exist forever 
● This can be changed in the incorporation document  
7) Free transferability of ownership interests 
● Applies to large corporations (usually have buy-sell agreements in small corporations) 
8) Access to capital 
● Can raise money and take the corporation public 
9) Attractive to talented employees  
B. Governing Law 
a. State Law 
i. Internal affairs doctrine 
o A conflict choice of law principle 
o All internal relationships for a corporation will be governed by the state of incorporation 
o This will not apply if have a dispute with a third party  
ii. MBCA = Model Business Corporations Act (also RMBCA = revised…) 
o Has been adopted in the vast majority of states 
o Small corporations usually incorporate in their local state and this will govern  
iii. DGCL = Delaware General Corporations Law 
o For larger corporations, the best course of action is to incorporate in Delaware and this 
will govern 
iv. Judge-made Corporate Governance Principles 
18
o Judge feel free to fill in the blank  
b. Federal Law 
● Applies to larger corporations 
● Federal securities law  
c. Stock exchange rules 
● Various rules that apply to corporations whose stock is traded on NYSE 
C. Ownership Structure 
▪ Ownership structure​ ​determines identity of officers, directors and shareholders, resulting in different 
mechanisms of control 
1) Closely Held Corporations 
● Small number of shareholders (owners) 
● No access to public securities markets (not freely transferable generally)  
● Typically, one person wears many hats 
● Shareholders normally exercise control 
● Often governed by prior shareholder agreements  
2) Public Corporations  
● Shares owned by large number of investors 
● Shares are traded in the public securities market 
● Separation of ownership and control 
1. Control is exercised by the board 
2. Daily operations are run by management  
3. Shareholders vote on directors  
4. Traditionally weak shareholder oversight  
● Process of going public: 
o Called an IPO (initial public offering) 
o File a Registration Statement (S-1) with SEC 
▪ All public filings are available on EDGAR 
3) Distinctions between the two  
● Public corporations are subject to federal disclosure requirements 
● Public corporations are subject to the threat of hostile takeover (“market for corporate control”) 
D. Corporate Social Responsibility 
▪ Duties to shareholders versus other stakeholders (creditors, employees, community) 
● US = corporation is run for benefit of shareholders 
● Europe = owe fiduciary duties to stakeholders 
o Stakeholders include employees and public 
E. Incorporation 
1) Draft Charter 
● Charter = articles of incorporation (MBCA) or certificate of incorporation (DGCL) 
i. Charter MUST contain  
o MBCA articles of incorporation must include: 
1. A corporate name, which must contain the word corporation, incorporated, 
company, or limited, or the abbreviation corp., inc., co., or ltd., or the like; and it 
must not be deceptively similar to name of another corporation   
2. The number of authorized shares, classes, and number for each class  
● If want to change later then must amend the charter 
3. Name and address of a registered agent and each incorporator  
o DGCL (same as above but has additional things) 

19
1. Business purpose (enough to state that “the purpose of the corporation is to 
engage in any lawful act or activity”). Acts outside of corporate purpose are void 
as ​ultra vires ​(beyond your power) 
2. Par value of stock (in some jurisdictions) (minimum value shares can be sold, 
usually nominal) 
ii. Charter SHOULD contain (doesn’t have to) 
1. Initial directors 
2. Corporate purpose 
3. Management provisions 
4. Bylaw provisions 
5. Director liability 
6. Indemnification  
2) Filing the articles of incorporation with the Secretary of State  
i. When can the State reject the filing of a charter? 
o State’s role is enabling not regulatory; cannot reject a charter where secretary feels some 
provisions are irrelevant or not authorized by law 
o Can reject: 
▪ Only if the name has already been taken 
▪ One of the required elements is missing 
▪ The purpose of the corporation is unlawful 
3) Differences in Charter and Bylaws 
i. Filing 
1. Charter is a public document filed with the state of incorporation. (Charter will trump 
Bylaws) 
2. Bylaws is a private doc. (usually very long and there is no filing) 
ii. Contents 
1. Charter contains only the basic info re corporation and is designed to protect public 
interest 
2. Bylaws contain rules of internal operations, including rules for shareholder and board 
meetings (when and where held, who calls the meetings; notice, quorum, and voting 
requirements); officer titles and duties, indemnification of directors and officers 
iii. Amendment Process (once amended called the second amended articles of incorporation) 
1. Charter is amended in a 3-step process 
1) Board adopts amendment and refers to shareholders; (They must have a meeting 
and everyone must have been notified before the meeting; they will pass a 
resolution saying what changes are being made to the bylaws) 
2) Shareholders approve; (Shareholders must vote to approve amendment of 
bylaws; they call shareholder meeting and must serve notice for meeting; Notice 
will include proxy materials (this has everything they will be voting on in 
shareholder meeting)) 
3) Approval filed with the state  
2. Bylaws may be amended by the shareholders directly, without the board prior approval 
4) Organizational Meeting  
● Decide who will fill what roles and their rank of authority, adopt bylaws, issue stock certificates  
● Grant v. Mitchell 
5) Liability for pre-incorporation activity 
i. Promoter: One who claims to act as an agent prior to incorporation. (person who is advancing 
the affairs of the corporation prior to incorporation) 
20
ii. Legal Issues: 
1. Does the corporation become a party to the contract? (the promoter signs the contract) 
▪ Yes, if contract adopted by corporation. (once the corporation is incorporated 
the corporation must adopt the contract to become a party) 
2. Is the promoter liable if the corporation breaches? (once incorporated promoter is 
protected) 
▪ Yes, unless company and investor release (promoter remains personally liable 
because signed deal prior to incorporation unless company and investor releases 
promoter and create Novation) 
3. Is the promoter liable if the corporation is not formed? (is there a way to hold 
corporation liable if they were never formed? Yes, look at liability with defective 
formation) 
▪ Yes, under MBCA § 2.04 
6) Liability with Defective Formation 
i. De Facto Incorporation 
o Treat improperly-incorporated entity as corp. if organizers:  
▪ Made a good faith, colorable attempt to incorporate, (this is the problem today, 
how do you make a good faith, colorable attempt? Everything is done online so 
it is hard to make this showing) 
▪ Had a legal right to do so, and (must have a statute that would allow you to 
incorporate) 
▪ Used the corporate privilege” (i.e. acted as if a corporation). 
ii. Corporation by Estoppel 
o Third party is estopped from denying the existence of the corporation if: 
▪ Third party thought firm was a corporation, and 
▪ A windfall if allowed to argue that firm was not corporation 
▪ Only allowed in contract cases 
F. Capital Structure ​(refers to all claims against the corporation) 
a. Equity​ = Stock (ownerships interest in the corporation) 
● Don’t get money until sell stock = liquidity  
● Issuing stock means to sell to shareholders 
i. Selling stock 
o Can sell on NYSE or NASDAQ 
o When a company is acquired their stock goes up because there is obviously something 
worthwhile in them because someone is willing to buy them (buyer willing to pay a 
premium price) 
ii. Charter 
1. Authorizes issuance of equity 
▪ Sets number of shares  
▪ Specifies classes and series (wants classes because want certain people to have 
control and the decision-making power and others be passive investors in it for 
the money) 
● Common 
o A series is Class A and Class B 
o If start with only common shares and later want to issue 
preferred must amend the articles of incorporation--to get around 
doing this can have blank check preferred shares in the original 
articles of incorporation 
21
● Preferred  
o Terms may be set out in a certificate of designations 
▪ This will set out all rights, preferences, and limitations  
o Blank check preferred shares 
▪ In articles of incorporation state the board of directors is 
authorized to issue preferred shares with the rights of 
which will be laid out in the certificate of designations 
▪ This is done to make it easier for the corporation to get 
money from investors (generally don't have to worry 
about the board going crazy with it but it is always a risk) 
▪ Set number of shares for each class 
2. Must specify characteristics of each class: 
▪ Preference 
● Preference in liquidation (only applies when corp. is liquidated) or 
dividends 
▪ Limitations 
● On voting rights or transferability  
▪ Relatives rights 
● Dividends 
● Liquidation rights 
● Voting rights 
o Some can vote their shares and others can't 
o MUST have one class that has voting rights 
● Conversion rights 
o Changing your interest from one class of stock to another class of 
stock 
o If allow will be in the agreement and it will have a ratio of what 
the conversion rate will be--usually limited to specific situations 
(not created as a matter of default) 
● Redemption rights 
o Right of corporation to buy back 
o Called call option--right to buy stock 
o The other type of option is a put option--right to sell stock 
o Must be spelled out in the charter  
● Preemptive rights 
o If the corporation offers to sell more stock, have the right to 
purchase the same amount of stock that you own 
iii. Authorized stock 
o The authorized amount is the max number of share the corporation is every authorized 
to issue (must amend charter to change the number of shares and/or classes)  
o Authorized, issued, and outstanding 
▪ Shares currently held by shareholders 
▪ Issued = stock sold to public or stockholders 
▪ Outstanding = Owned by stockholders 
▪ Outstanding shares are the only ones that can be utilized to enforce rights  
o Authorized, and unissued 
▪ Available for sale 
o Authorized, issued, and not outstanding  
22
▪ These are held by the corporation 
▪ Generally, shares the corporation has bought back  
▪ In DE, called treasury shares 
▪ Under MBCA, no treasury shares 
● The shares bought back go back into pool of authorized but unissued 
shares  
iv. Common Shares 
o This is the default category of stock 
o Unlimited voting rights 
o Right to residual assets of the corporation  
v. Preferred Shares 
o Have preference (priority) over common shares 
o Will always get paid first, so before the common stock holders 
o Preferred shareholders stand before common shareholders in dividends and liquidation 
o May be cumulative or noncumulative 
o May be participating or nonparticipating (straight) 
b. Debt ​= a promise to repay on a certain date, called the maturity date, and to pay interest in regular 
installments 
● Anytime going to sell debt, always possible may have to register with SEC if can’t find 
exemption  
i. Types of debt 
1. Notes (shorter term) 
▪ Usually shorter than 9 months 
▪ Not subject to securities regulation 
2. Bonds (secured) 
a. Terms laid out in an indenture (specific term of art used to describe the K 
between the Corporation, bondholder, & trustee; very specific contract that adds 
an intermediary) 
i. Indenture is a contract between the corporation and a trustee. (trustee 
acts on behalf of the bondholder; the trustee enforces the bondholder's 
rights) 
ii. Indenture describes procedures for bond issuance, payment, redemption 
and discharge. (repayment – repay entire upon expiration) 
o Redemption – allows to call the bonds 
▪ Call = option to buy 
▪ Works as a refinance – issue new bonds to receive money 
to use to buy back old bonds (will have the same principal 
balance but the interest most likely is lower) 
o Priority – repayment hierarchy in realtion to other debt 
o Conversion – can covert debt securities into capital stock 
▪ Gives the bond holders the option to call 
▪ So they will turn in the bonds and then get the stock in 
return 
iii. Covenants = promises by the corporation to perform certain actions  
o Promise to pay on monthly basis, promise to let bank know if file 
bankruptcy 
o There are positive and negative covenants 
▪ Negative keeps you from doing something 
23
iv. Events of default, which allows bondholders to accelerate payment. 
o Event of non-repayment 
o Missing 3 payments in a row, filing bankruptcy 
o Cross-default--sister entity defaults 
o Default on revolver (can take money out but must keep paying it 
off to take more out) 
b. Types of Bonds 
i. Registered 
o Holder’s name and address listed in registry  
ii. Bearer 
o Not attached to a specific name; as good as cash 
▪ Has coupons attached and just need to have coupon; if 
holder of coupon can exercise any of the rights 
c. Bond Ratings 
● Reflect credit-worthiness of the bonds 
● Used by investment professionals to assess likelihood of debt repayment 
● Investment grade (IG) bonds=likely to meet payment obligations. Banks 
are allowed to invest. (they are safe from mutual bonds) (If decrease 
someone's debt they just are dropped in category because there is a 
higher risk now) 
● High-yield (“junk”) bonds​ =​ junk bonds (highly subordinated securities) 
(very risky) 
● Government bonds = if want to invest with no risk; considered to be 
very safe because US has never defaulted; low interest rate 
● Risk is proportionate to the return 
3. Debentures (unsecured) 
a. Same as bonds except no security interest 
c. Priority Scheme 
1) Secured (collateralized) bondholders 
2) Unsecured debenture holders 
o Second person gets paid nothing until the first person has been paid in full 
3) Holders of subordinated debt 
4) Preferred stockholders 
5) Common stockholders 
● Subordinated debt –bank knows corp. is not doing well and will probably default and they go to 
bank and say I have an investor that will give me some money but they want priority over the 
bank 
o Bank will be willing to do this because if they don't and the corp. doesn't have money, 
then the bank isn't going to get anything anyway 
● Inter-creditor agreement -- will trump any other agreement and now bank is second so are 
subordinated 
d. Benefits of Issuing Debt 
i. Interest ​payments are deductible (dividends are not) (interest rate on a loan is much lower than 
anything would pay back through equity) 
ii. Debt ​is cheaper (reflects lower risk level) 
o Leverage ​(using borrowed money to generate returns greater than the cost of borrowing) 
(based on fact that debt is cheaper because it is less risky; so borrow money from lender 
and use the money to do something riskier hoping for a higher return) 
24
iii. No ​control rights (stockholders get to vote while bondholders don't get to vote on anything) 
iv. No f​ iduciary duties (no one owes any fiduciary duties to bondholders unless the corporation is 
insolvent; all bondholders rights are contractual) 
v. No ​statutory limit on debt (not one on equity either but for equity have to go through charter 
amendment process but bonds are not mentioned in the charter at all) 
vi. Some​ institutional investors can’t invest in equity (pension fund or life insurance co.) 
e. Ways Creditors (bondholders) can protect themselves 
i. Security/collateral 
ii. Personal ​guarantee (the individual backs the loan instead of the corporation) 
iii. Requiring ​a particular level of liquidity or equity capitalization (liquidity is cash on hand; bank 
may require that you keep a certain amount of money in the corp. account at all times) 
iv. Requiring ​regular financial reports (financial statements; want a reporting of how well doing so if 
not doing well, they can stop them before it is too late) 
v. Requiring ​approval for certain transactions 
vi. Definition ​of default and acceleration rights (acceleration rights are where the bank can 
accelerate your payment schedule if you default (for example: if you miss 3 interest payments in 
a row then the entire amount due is due immediately)) 
f. Grimes v. Alteon 
● Board of directors has “​exclusive authority to govern and regulate the corporation’s capital 
structure​.” 
G. Directors and Shareholders 
a. Management of a Corporation 

 
b. Control 
● Shareholder v. Management 
o Separation of ownership and control 
o Possible conflict of interest 
o Agency costs 
o Shareholders want to increase share value 
o Management may be more interested in job security, luxury, and higher bonuses 
c. Directors 
i. Role of the Board 
o A corporation MUST have a board 
▪ But shareholders in closely held corps can do away with board by so noting in 
charter 
o Exact number or range MUST be specified in the Charter or Bylaws 
▪ Best to put in the Bylaws because it is easier than Charter to amend 
25
▪ May chose Charter because shareholders have control over changing it 
o To manage (or supervise management of) the corporation 
▪ Delegate power to officers (Officers powers generally contractual or in Bylaws) 
▪ Have a limited role except in times of crisis 
ii. Inside v. outside Directors 
o Inside Directors 
▪ Otherwise employed by the company 
▪ All the highest-level officers are generally on the board (CFO, COO, head legal 
counsel) 
o Outside Directors 
a. Independent 
● Don’t have financial ties to the company 
● Majority – NYSE requires most directors be outside, independent 
directors 
o This is to help avoid conflicts of interest 
▪ So, when faced with a decision on whether to sell the 
company or not, are not tainted with a conflict of interest 
b. Not Independent 
● Has a financial stake in the company  
iii. Terms of Office 
o Elected by shareholders at an annual shareholders’ meeting (​proxy statements go out 
before a meeting and list those currently on the board and by default they are reelected if 
don't respond to proxy statement; proxy battle because insurgent will put together their 
own slate of directors) 
▪ Unless: 
● A ​vacancy occurs midterm 
o Vacancies ​may occur because of resignation, death, removal of 
director, or by the creation of a new spot 
o Directors ​may resign at any time by delivering written notice 
o Vacancies ​may be filled by directors or shareholders.  
▪ directors can fill the vacancies in the interim up until the 
next shareholder meeting 
● Directors’ ​terms are staggered 
o Staggered (or classified) board is​ ​divided into several classes; 
election is for a certain term, majority of directors continuing 
without the need for re-election. (could break them up into 3 
classes with 3 directors each (have 9-person board) Class A that is 
replaced year 1, Class B that is replaced year 2, Class C that is 
replaced year 3) (true purpose of staggered board is to prevent 
proxy fights) 
o Anti-takeover measure. (the insurgent would have to win the vote 
two years in a row to establish a majority) 
o Can be removed by shareholders, with or without cause 
a. Unless ​the charter provides that directors may be removed only for cause, or 
(this is done to encourage people to serve on the board because it hurts a 
person's reputation if they are removed from the board even if it was without 
cause) 
b. If ​the board is staggered (then under DGCL removal is for cause) 
26
iv. Directors Actions 
1. Duly Called Meeting 
▪ Annual Meeting 
o Specified in Bylaws 
▪ Special Meeting 
o Must ​send a notice of meeting stating date, time, and place 
▪ Don’t need to include purpose unless Charter or Bylaws require it 
▪ Director may waive notice of a meeting either in writing or by 
showing up at the meeting 
o Must ​give enough time for them to be able to attend 
▪ May be present remotely (by phone) 
▪ Quorum must be present 
o Quorum = Majority of directors (Charter or Bylaws may specify 
otherwise) 
▪ Vote 
o Majority of directors present (can alter in Charter or Bylaws) 
2. Unanimous written consent 
▪ A way for directors to act without holding a meeting (​unless charter or bylaws 
provide otherwise) 
▪ Directors may convene over phone call or distribute document among them that 
everyone signs -- ​must be unanimous 
▪ 1 ​page resolution with a signature by every person on the board 
▪ Some ​companies allow electronic signature or a stamp (depends on companies' 
rules) 
3. Action by a committee 
▪ Can ​do certain things as a committee and the acts of the committee will be 
deemed to be the actions of the board  
▪ Committee types:  
● Audit committee (financial controls and risk assessment); 
● Nominating committee (nominates directors) 
● Compensation committee. 
▪ Limitations on committee actions:  
● Cannot delegate actions that will need shareholder vote 
● Committee cannot adopt, amend, or repeal bylaws 
● MBCA: committee cannot fill board vacancies or authorize dividends, 
except pursuant to a set formula   

27
●  
d. Shareholders 
i. Voting 
1. 1 share = 1 vote​ (​Default Rule​)(count shares not heads because 1 shareholder can have 
1 million shares) 
2. Shareholder can vote on: 
a. Election of directors 
b. Fundamental transactions 
i. Amending charter 
ii. Amending bylaws 
iii. Approving a merger 
iv. Approving the sale of assets not in the ordinary course of business 
v. Approving dissolution of the company 
vi. Ratifying conflict-of-interest transaction 
3. Quorum: 
a. Shareholders holding a majority of shares must be present  
● Can modify in Charter or Bylaws 
b. Once have established a quorum need the majority of those present to pass the 
action 
c. Plurality vote elects directors, otherwise majority wins 
d. Can vote in person or by proxy 
● Proxy voting (Can ​delegate their vote to someone else) 
o A ​proxy is authorization given by a shareholder to another 
person to vote shareholder’s shares. (representative picked is a 
proxy for shareholder because they gave rep. a proxy through a 
proxy statement) 
▪ Proxy form MUST be filed, sent to corporation.  
▪ Proxy ​holder is shareholder’s agent, subject to 
shareholder control and owing fiduciary duties to 
shareholder. (shareholder can give specific instructions on 
how to vote or say you can have my proxy and vote 
however you want) 
4. Shareholder Action 
a. Shareholder meetings 
i. Annual (must be held; unless directors are elected by written 
consent—unusual; if not held shareholders can get judicial order setting a 
date for the meeting) 
o Director elections 
▪ Takes place at an annual meeting 
▪ Most are uncontested, ​simply engendering a “vote of 
confidence” for nominees proposed by the nominating 
committee​ Record date for meetings 
▪ Default rule​: straight voting (shareholders vote all shares 
for each open seat) 
● Majority wins every seat 
● If person has 100 shares and 9 open seats, can 
vote 100 shares on each seat 

28
▪ Cumulative voting allows shareholders to concentrate 
voting power by cumulating votes (number of shares 
times number of directors to elect) 
● Ensures minority shareholders have voice on 
board 
● If person has 100 shares and 9 open seats, can 
cast 100 shares per seat or 900 shares for 1 seat 
● Must opt into this voting in Charter or Bylaws 
● More common in closely-held corporations  
o Record date for meetings 
▪ Who gets to vote? 
● Those who own shares on the record date  
▪ The Board sets the record date 
▪ MBCA = can’t be more than 70 days before meeting 
▪ DGCL = can’t be more than 60 days before meeting, nor 
less than 10 days before meeting 
ii. Special 
o May be called by board or shareholders (if shareholders are given 
this power in the charter or bylaws) 
o Must give notice (time, place, and ​PURPOSE​) 
▪ Process for calling special meeting and for deciding time 
and place outlined in Charter or Bylaws 
▪ If notice is insufficient, ​actions taken at the meeting are 
voidable by shareholders who did not attend 
▪ Attendance = waiver of right to contest improper notice 
o MBCA – Special meetings may be called by board or 10% 
shareholder 
o Adlerstein v. Wertheimer 
b. Written consent (unless Charter disallows) 
● MBCA = requires unanimous consent 
o This power is not very important because of this requirement and 
rarely can get unanimous consent  
● DGCL = same number as what is required at a meeting 
o This power is extremely useful to shareholders or to an entity 
seeking to take control of another company, since it means a 
majority coalition of shareholders can act, such as to replace the 
board, without having to ask the board to call a special meeting, 
set a meeting date, and notify shareholders of the special meeting 
▪ A majority coalition of shareholders can act without being 
subject to the board's tactical decisions to delay calling a 
special meeting 
● Designed to make action by shareholders in closely held corporations 
easier 
H. Dividends and Distributions 
a. Intro to Financial Statements  
i. Balance Sheet 
o This snapshot of how a corporation is doing at any given moment in time 
o Just looking at current affairs of the corporation  
29
ii. Income Statement 
o Net income over a period of time 
o Most are prepared for the year 
o Bottom line 
o Total revenues (total amount of money bringing into the corporation) - all expenses = 
net income (net profit) 
iii. Statement of Cash Flows 
o A statement summary reflecting cash generated over period of time and cash used during 
that same time period 
o Where did money come from and where did it go 
▪ Important to know where money came from because they want to know if the 
money is from profits or is it from a lender 
▪ Just because a company has 10 million dollars in its bank account, does not mean 
it is doing well. That money could be from a loan 
ii. All are available on EDGAR from SEC (forms 10K and 10Q) 
b. D​ividends 

 
c. Dividend Issuance and Contractual Rights 

 
d. Board Discretion in Dividend Issuance 
● Board Discretion 
o DGCL = directors of every corp. may declare and pay dividends upon shares of its 
capital stock 
o MBCA = directors may authorize and corporation may make distributions to its 
shareholders 
30
o In all circumstances, courts are deferential to the board’s decision to declare a dividend 
▪ DGCL and MBCA are both very permissive 
● Board has power to declare dividend but doesn’t have to declare any  
o A shareholder can never sue to enforce dividends; well can sue but will lose 
o No​ ​one can force a corporation to declare a dividend but once a dividend is 
declared, then can sue for breach of contract if not paid 
● Dodge v. Ford Motor Co​.  
e. Dividend Preference 
● Common vs. Preferred 
1. Preferred Stock 
▪ Has preference over common stock with respect to dividends and liquidation 
▪ Is paid first – not contractual and isn't paid out until board chooses to pay a 
dividend 
● Once board declares a dividend then it becomes contractual 
o Requires a meeting and a resolution must be passed saying 
dividend will be paid 
o Record date is set and whoever is a stockholder on that date gets 
paid 
o They will also set out the distribution date 
▪ Preferred stock holder’s rights are contractual 
● Even if says in charter they will get $1 per share annual dividend, they 
can’t sue if don’t get it because dividends are discretionary and the right 
isn't contractual until the dividend is declared 
▪ Preferred holders can contract to be participating so don't get less than common 
holders 
2. Participating and Cumulative Preferred Stock 
▪ Participating  
● Paid out before the common stock because preferred 
● Get what they are contractually entitled to plus the same dividend as the 
common stock holders 
▪ Cumulative 
● Can’t pay a penny to common stock until all arrears on preferred stock 
are paid 
o Arrears = payments on which you are late 
● Even though hasn't paid for years can't sue over nonpayment until 
company declares a dividend 
● Possible for a company to be in arrears infinitely and will never pay out a 
dividend to the common because can't cover their arrears 
● Preference stripping merger  
o A company creates a subsidiary and makes all those common 
stockholders the stockholders of the subsidiary and the subsidiary 
pays out dividends 
▪ This around issue of not being able to cover the arrears 
o If a preferred stock in arrears and sold then arrears transfers with 
stock 
3. Common Stock 
▪ Always get the most (they see the residual) 
 
31
 

 
f. Restrictions on Distributions 

 
i. Insolvency tests = MBCA (​Both tests need to be met to pay out a distribution​) 
1. Solvency Test 
▪ Can a corporation meet all its debts 
▪ Equitable insolvency -- court of equity 
▪ Can't just have assets, must have liquidity 
32
▪ Cannot make a distribution would result in company’s inability to pay debts 
when due 
▪ Fraudulent conveyance problem 
2. Balance Sheet Test 
▪ If liabilities more than assets, can't pay dividends 
▪ Simply looks at the balance sheet 
▪ Prohibits distributions would result in total assets being insufficient to pay sum 
of corporation’s liabilities and any liquidation preferences 
ii. Impairment of Capital Test = Delaware 

 
o The corporation must keep some capital on hand for the protections of the creditors 
o Still in DE and NY 
o Comes from notion of par value 
o The legal capital is always off limits for dividends 
▪ Can​ ​never pay out a dividend from legal capital 
▪ Under DE law the only thing that can't be touched is legal capital 
o Dividends must be paid out of the Surplus (capital or earned) 
o If forget to list par value in charter, then the entire amount will become legal capital and 
none of it can be touched 
o Legal Capital and Capital surplus are tied to specific events (like selling stock) 
o So Earned Surplus or Retained Earnings is the one that will be adjusted up or down 
o Impairment of Capital Test:  
▪ A corporation issues 10 shares of $100 par-value stock. (Here can't pay out any 
dividends because assets - Liabilities - Legal Capital = 0) 
▪ Assets $1000 cash Liabilities $0 
Capital Accounts 
Legal Capital $1000 (Shareholder equity is broken down into legal capital, capital surplus, and earned surplus) 
Capital Surplus $0 
Total $1000 
▪ What if par value is $1. Company issues 10 shares for $100 each. 
▪ Assets $1000 cash Liabilities $0 
Capital Accounts 
Legal Capital $10 
Capital Surplus $990 (any amount in excess of par value when sold stock to investors) (corp. could pay this out to 
stockholders, but would be stupid and no one would do 
this) 

33
o No par Stock 
▪ A corporation issues 10 shares of no-par stock, raising $1000. ​See ​DGCL 154. 
▪ Assets $1000 cash Liabilities $0 
Capital Accounts 
Stated Capital $100 
Capital Surplus $900 
Total $1000 
o Earned Surplus  
▪ Company issues 10 shares of $1 par-value stock for $100 each. The same 
corporation made $2000 in profits.   
▪ Earned surplus is the same as retained earnings in accounting. 
▪ Assets $3000 cash Liabilities $0 
Capital Accounts 
Legal Capital $10 
Capital Surplus $990 
Earned Surplus $2000 (if had a 2,000 loan = liabilities then this earned surplus will become a negative number) 
o Klang v. SFD, Inc.​   
▪ Directors have latitude to depart from the balance sheet to calculate surplus 
g. Creditor Protections 
i. Director Liability for Unlawful Dividends 
1. RMBCA = A director who votes for or assents to a distribution in excess of what may 
be authorized and made pursuant to section 6.40(a) or 14.09(a) is personally liable to the 
corporation for the amount of the distribution that ​exceeds what could have been 
distributed​ without violating section 6.40(a) or 14.09(a) if the party asserting liability 
establishes that when taking the action the director did not comply with section 8.30 
2. DGCL = In case of any willful or negligent violation of the provisions of sections 160 or 
173 of this title, the directors under whose administration the same may happen shall be 
jointly and severally liable at any time within six years after paying such unlawful 
dividend or after such unlawful stock purchase or redemption, to the corporation, and to 
its creditors in the event of dissolution or insolvency, ​to the full amount 
▪ If authorize a dividend that should not be paid out, will be held personally 
liable 
ii. Exceptions to Director Liability 
1. DGCL 172: Directors can rely on determinations of corporate accountants when 
deciding whether capital impairment test is met 
2. DGCL 174(c): Any director can sue stockholders who received dividends knowing it was 
illegal 
3. DGCL – In addition to the matters required to be set forth in the certificate of 
incorporation by subsection (a) the certificate of incorporation may also contain any or 
all of the following matters -- ​A provision eliminating or limiting the personal 
liability of a director​ to the corporation or its stockholders for monetary damages for 
breach of fiduciary duty as a director, provided that such provision shall not eliminate or 
limit the liability of a director (iii) under section 174 
iii. Fraudulent Transfer Law 
o Transfers of property for purpose of defrauding creditors 
o Actual fraud​ ​(intent or close relationship between transferor and transferee) 
o Constructive fraud (adequacy of consideration and financial position of the debtor) 

34
▪ Transferor receives less than reasonably equivalent value for the transferred 
assets 
▪ Seller was insolvent or became insolvent because of the transfer 
I. Limited Liability and Piercing the Corporate Veil 
a. Benefits of Limited Liability  
● Enables shareholders to diversify more efficiently 
● Permits free transferability of shares 
o May inherent some lawsuits at some point 
● Reduces monitoring costs 
o Shareholders always have reason to monitor what is happening 
b. Effects of Limited liability  
● It increases the cost of debt and decreases the cost of equity to the corporation 
● Major potential social cost: 
o people who have limited liability have an incentive to engage in riskier than optimal 
activities because they are not forced to bear the total costs of such behavior 
▪ This increased risk = moral hazard 
● Moral hazard doesn’t impose social costs on transactions involving the 
corp. and voluntary creditors 
● It does impose social cost on transactions involving involuntary creditors 
o Special problems arise in trying to justify limited liability within corporate groups, such as 
a parent company and its wholly owned subsidiaries 
▪ The separate legal status of parent and subsidiary corporations is respected in 
most cases, and whole enterprise theory of liability for corporate groups in the 
parent/subsidiary context has been soundly rejected 
▪ However, sometimes courts will pierce the veil and require shareholders to pay 
the entire amount of a contract or a judgment on a tort law claim, beyond the 
amount of the shareholder's investments 
c. Piercing the Corporate Veil (look to LLC for the analysis and factors) 
● Direct Liability  
o First – whether liability may attach to the shareholder directly by reason of shareholder's 
own actions instead of recognizing the liability as a corporate liability that must be paid 
by the shareholder personally because of the piercing analysis 
● Corporate Formalities 
o Second -- If no personal liability, whether corporate formalities have been carefully 
observed 
▪ Corporation has separate bank account, no commingling of personal & 
corporate funds, regular meetings of directors and shareholders 
▪ If formalities have been followed, few courts will pierce 
● Fairness 
o Third most courts require showing injustice or unfairness to link the wrongdoing to the 
harm 
▪ Two types of injustice are most common: 
i. Where disregard of corporate entity has been visible to third party and 
third party has reason to be confused if were dealing with corporation or 
an individual 
ii. Where the shareholder has disregarded the separateness of the 
corporation's funds and treated them as her own 
● Soerries v. Danclause 
35
o Failure to follow corporate formalities (alter ego theory) + some element of injustice 
o Limited liability shield will not protect you from your own torts  
 
 
   

36
Closely Held Corporations 
A. Shareholder Agreements 
a. Purpose 
● To protect minority shareholder (makes a corporation more attractive to investors). Shareholder 
agreement binds all investors. 
b. Types 
i. Vote pooling agreements 
o Two or more shareholders may provide for the manner in which they will vote their 
shares by signing an agreement for that purpose 
▪ A voting agreement created under this section is specifically enforceable 
o This agreement obligates shareholders to vote together as a block  
o variety of ways to accomplish it 
▪ DON’T leave it as an agreement to agree 
▪ instead - appoint an independent 3rd party that can resolve a deadlock 
between them 
▪ if one investor has more business acumen - they can agree to give that dude 
an irrevocable proxy 
ii. Agreements purporting to control board actions  
o Minority shareholders can bargain for certain rights 
▪ Affirmative Rights 
● Right​ ​to name corporate officers 
● Right to employment 
● Right to a certain salary 
● Right to a dividend 
1. not a guarantee  
▪ Negative Covenants  
● Right to veto certain corporate transactions 
c. Enforceability 
i. Traditional rule 
o McQuade v. Stoneham 
▪ The ​stockholders may not, by agreement among themselves, control the 
directors​ in the exercise of the judgment vested in them by virtue of their office 
to elect officers and fix salaries. Their motives may not be questioned so long as 
their acts are legal. The bad faith or the improper motives of the parties does 
not change the rule. Directors may not by agreements entered into as 
stockholders abrogate their independent judgment 
ii. Modern Rule 
o Today, can pretty much agree to anything you want 
o MBCA 7.32: An agreement among the shareholders is effective even though it is 
inconsistent with one or more other provisions of this Act in that it: 
▪ eliminates the board of directors or restricts the discretion or powers of the 
board​;  
▪ governs ​authorization or making of distributions​ whether or not in 
proportion to ownership of shares, subject to limitations in section 6.40; (even 
though board has discretion to pay out dividends can contract for dividends to 
be paid but courts will say final say is with board) 
▪ ​establishes who shall be directors or​ ​officers​ of corporation, or their terms 
of office or manner of selection or removal; 
37
▪ governs, in general or in regard to specific matters, exercise or division of ​voting 
power​ by or between shareholders and directors or by or among any of them, 
including use of weighted voting rights or director proxies; 
▪ establishes terms and conditions of any agreement for ​transfer or use of 
property​ or provision of services between corporation and any shareholder, 
director, officer or employee of corporation or among any of them; 
▪ transfers to one or more shareholders or other persons all or part of ​authority 
to exercise corporate powers or to manage business and affairs of 
corporation​, including resolution of any issue about which a deadlock exists 
among directors or shareholders; 
▪ requires ​dissolution​ of corporation at request of one or more of shareholders or 
upon occurrence of a specified event or contingency; or 
▪ otherwise governs exercise of corporate powers or management of business and 
affairs of corporation or relationship among shareholders, directors and 
corporation, or among any of them, and is not contrary to public policy. (Very 
broad -- basically you can agree to anything, rules can be modified in any way 
they want to) 
o An agreement authorized by this section shall be: 
▪ Set ​set forth (A) in the ​articles of incorporation or bylaws ​and ​approved by 
all persons who are shareholders ​at the time of the agreement or (B) in a 
written agreement​ that is signed by all persons who are shareholders at the time 
of the agreement and is made known to the corporation; 
▪ subject to ​amendment only by all persons who are shareholders at the time 
of the amendment​, unless the agreement provides otherwise; and (once agree to 
something and want to change it, need approval of everyone) 
▪ ​valid for 10 years​, unless the agreement provides otherwise. (this is from when 
courts weren't as accepting of shareholder agreements, not so much a issue 
today) 
▪ The existence of an agreement authorized by this section shall be ​noted 
conspicuously on the front or back of each certificate​ for outstanding shares 
. . . . (when have a stock certificate need to have notice of the agreement 
indicated on the stock certificate, this is to give notice to third parties about the 
shareholder agreement) 
o Modern analysis – ​Ronnen v. Ajax Electric Motor Corp. 
d. Formalities  
● Must always be concerned with formalities here 
● Set forth in​ ​Charter or Bylaws or in a written agreement, the existence of which is noted on 
stock certificate 
● Valid for 10 years unless the agreement provides otherwise  
B. Irrevocable Proxies  

38
 
C. Transfer Restrictions (restraints on alienation) 
a. Purpose 
● Default Rule​: freely transferable 
● Control the selection of business associates 
● Provide certainty in estate planning 
● Ensure corporation complies with closely held corporations statutes (S Corp regulations or 
Securities Act exemptions) 
o When want the S corporation tax benefits, must be US citizens and if have too many 
stockholders won't qualify 
o With 3 owners probably exempted from registering with the SEC (private placement 
exemption) 
b. Formalities 
● Charter, Bylaws, or separate agreement 
o The articles of incorporation, bylaws, an agreement among shareholders, or an 
agreement between shareholders and the corporation may impose restrictions on the 
transfer or registration of transfer of shares of the corporation 
● Note restriction on stock certificate 
o A restriction on the transfer . . . is valid and enforceable against the holder or a transferee 
of the holder if the restriction is authorized by this section and its existence is ​noted 
conspicuously on the front or back of the certificate​…​Unless so noted, a 
restriction is not enforceable​ against a person without knowledge of the restriction 
c. Types 
● Options 
o Allows corporation an option to purchase any shares if someone wants to sell 
● Buy-Sell agreements 
o If someone wants to sell, the corporation is obligated to buy them 
▪ Different from option because the corporation is mandated to buy (it is not 
optional) 
o Provide liquidity for shareholders who wish to withdraw 
o Determine the price of the shares at a time when none of the parties to the agreement 
knows which of them will be the sellers and which will be purchasers 
o Allow the principals of the corporation to plan with certainty 
o Forms (how prices are determined) 

39
▪ Fixed prices (must be updated constantly to reflect the current value of the 
shares) 
▪ Book value (most popular measure because of ease of determination, but it is 
based on historical costs and may not reflect true underlying values) 
▪ Appraisal (has potential to be very good, but parties should decide beforehand 
on what basis the business should be appraised) 
▪ Formula (which suffers from being very complicated) 
● Prior-Approval or consent  
o The corporation can vote to decide if you can sell your shares 
▪ Court are suspicious of this because allows the corporation to discriminate 
against shareholders (gives corporation too much discretion) 
2. Normally enforceable as long as approval may not be unreasonably withheld 
3. Can decided who grants the consent (shareholders, directors, or corporation) 
● Transfer prohibitions  
o Overall statement that not allowed to transfer at all 
o Likely unenforceable unless coupled with some other mechanism 
o Viewed very skeptically by courts and usually would be struck down as unreasonable 
d. Enforceability 
● Test for validity​ of transfer restrictions 
1. Restrictions must comply with formal requirements and must be conspicuously noted on 
the stock certificates. DGCL 202(a); MBCA 6.27(a) and (b). 
▪ No restriction can affect shares issued before the restriction is adopted unless the 
holders vote in favor of the restriction. DGCL 202(b); MBCA 6.27(a). 
2. Restrictions must be for a proper purpose. Test is reasonableness. 
▪ Valid if reasonably necessary to advance the corporation’s welfare or attain the 
objectives set forth in the corporation’s charter. 
▪ Factors courts will consider in determining reasonableness of transfer restrictions 
● Size of the corporation (a small size and unique capital structure will 
matter) 
● Degree of restraint on alienation (absolute prohibition not reasonable 
and prior consent are probably unreasonable) 
● Duration of the restriction (will this last forever or only 1-2 years) 
● Method of determining the transfer price of shares (objective methods 
are viewed more favorably) 
● Likelihood of restrictions working in favor of attaining corporate goals 
(must give valid reasons for stock restrictions) 
● Possibility that a hostile stockholder would injure the corporation 
● Odds that the restriction would benefit the corporation (once given corp. 
purpose, how likely that this restriction will help attain this result?) 
● Whether the restriction is the result of fraud or breach of fiduciary duty 
▪ This is a highly deferential Standard, so whatever the board agrees on is 
fine so long as some reasoning 
● Capital Group Companies, Inc. v. Armour 
D. Voting Trusts ​(courts view these with suspicion) - reallocates control in a closely held corporation 
a. Formalities  
● Were created to overcome rules against irrevocable proxies 
● One or more shareholders may create a voting trust, ​conferring on a trustee the right to vote 
or otherwise act for them, by ​signing an agreement ​setting out the provisions of the trust 
40
(which may include anything consistent with its purpose) and ​transferring their shares ​to the 
trustee  
● A voting trust becomes effective on the date the first shares subject to the trust are registered in 
the trustee's name. A voting trust is ​valid for not more than 10 years ​after its effective date 
unless extended under subsection (c) 
● How is this different from vote pooling agreement? 
o To create a voting trust must appoint a trustee and transfer legal title to trustee and true 
owners entitled to all benefits of stock but cannot vote on shares (beneficial ownership 
certificates = what true owner retains) 
o Voting trust not valid for more than 10 years; goes to court's distrust of formation of 
these trusts 
● If an agreement is determined to be a de facto voting trust, the agreement inevitably will have 
failed to comply with the formal requirements of voting trusts and will be invalid 
b. Purposes for Voting Trust ​(how to justify the creation of a trust) 
● To aid in reorganization plans and adjustments with creditors in bankruptcy or financial 
difficulty 
● To assist financing, to procure loans, and to protect bondholders and preferred shareholders 
● To accomplish some definite plan or policy for the benefit of the company and to assure 
stability and continuity of management for this purpose 
● To prevent rival concerns or competitors from gaining control 
● To apportion representation and protect minority interests by putting directors’ selection in 
impartial hands 
● In connection with mergers, consolidations, or purchases of a business to assure that 
predecessors or constituents have representation even though in the minority 
c. Improper Voting Trust Puposes (​can't specifically say these but can have them in mind and just say a 
legitimate corporate policy) 
● To establish alien or minority control for a long period of time with restrained discretionary 
powers and without definite obligations or beneficial purposes 
o Such as for the individual benefit of promoters and investment bankers 
● To secure employment, salaries, contracts, and other individual benefits from a controlled 
management, or 
● To freeze the control of a group in office as an end to itself 
d. Enforceability 
● Modern view enforceable unless contrary to public policy  
e. Warehime v. Warehime 
● Valid for not more than 10 years 
E. Classified Shares-know for test 
a. Purpose 
● To allocate control among various classes of stock  
b. Types 
● Giving​ ​a class veto power over all decisions 
● Giving a class no voting power 
● Allowing a class vote only on certain matters 
● Providing a class the right to board representation, etc. 
c. Formalities  
● Rights, preferences and limitations must be specified in the charter 
o Delaware interprets corporate charters as ordinary contracts 

41
▪ When interpreting terms of preferred stock, De. courts employ a special rule of 
interpretation: “any rights, preferences and limitations of preferred stock that 
distinguish that stock from common stock must be expressly and clearly stated 
[and] will not be presumed or implied.” ​Benchmark Capital Partners v. Vague 
d. MBCA §6.01 Authorized Shares 
a) The articles of incorporation must set forth any ​classes​ of shares and ​series ​of shares within a 
class, and the ​number of shares of each class and series​, that the corporation is authorized to 
issue. If more than one class of shares or series of shares is authorized, the articles of 
incorporation must prescribe a ​distinguishing designation ​for each class or series and must 
describe, prior to the issuance of shares of a class or series, the ​terms, including the 
preferences, rights, and limitations​, of that class or series. Except to the extent varied as 
permitted by this section, all shares of a class or series must have terms, including preferences, 
rights and limitations, that are ​identical with those of other shares of the same class or 
series​. (What are the preferences, rights and limitations? This is where 601 B & C come in) 
b) The articles of incorporation ​must authorize ​(1) one or more classes of shares that together 
have ​unlimited voting rights​, and (2) one or more classes of shares (which may be the same 
class or classes as those with voting rights) that together are entitled to receive the ​net assets ​of 
the corporation upon dissolution. (Must be very specific with respect to voting rights) 
c) The articles of incorporation ​may authorize ​one or more classes or series of shares that: 
1. have special, conditional, ​or limited voting rights, or no right to vote​, except to the 
extent otherwise provided by this Act; (ok as long as one class has unlimited voting 
power) 
2. are ​redeemable or convertible ​as specified in the articles of incorporation: 
i. at the option of the corporation, the shareholder, or another person or upon the 
occurrence of a specified event; 
ii. for cash, indebtedness, securities, or other property; and 
iii. at prices and in amounts specified, or determined in accordance with a formula; 
3. entitle the holders to ​distributions ​calculated in any manner, including dividends that 
may be cumulative, noncumulative, or partially cumulative; (dividends are still 
discretionary; cumulative preferred stock -- the dividends don't go away the years the 
dividends are not declared, they just go into arrears; must pay off those in arrears before 
can pay out on the common) 
4. have ​preference ​over any other class of shares ​with respect to distributions​, including 
dividends and distributions upon the dissolution of the corporation. 
d) Terms of shares may be made dependent upon facts objectively ascertainable outside the 
articles of incorporation in accordance with section 1.20(k). 
e) Any of the terms of shares may vary among holders of the same class or series so long as such 
variations are expressly set forth in the articles of incorporation​. 
f) The description of the preferences, rights and limitations of classes or series of shares in 
subsection (c) is not exhaustive.  
F. Cumulative Voting 
▪ Default rule: straight voting 
i. position 1 on board of directors:  
ii. position 2 
iii. position 3 
▪ Method of counting shareholder votes in director elections in which each shareholder is entitled to cast 
a number of votes equal to the product of the number of such shareholder’s shares times the number of 
directors to be elected 
42
▪ Under cumulative voting a shareholder may cast all of his or her votes in favor of a single director rather 
than allocating them among the candidates  
▪ Purpose 
● to increase minority participation on the board 
▪ Formalities  
● “opt-in” in charter. MBCA 
▪ Formula: 
● [shares voting/(directors to be elected + 1)] + 1 = 1 shares required 
● Example: 3 directors to be elected and 1800 outstanding shares of stock outstanding  
o For 1 director 1,800/4+1 = 451 
o For 2 directors 3,600/4+1 = 901 
o For 3 directors 5,400/4+1 = 1351 
▪ Staggering the board may defeat the effect of cumulative voting 
● If stagger the 3 directors to terms of every 3 years, then each year vote on one director 
o 1,800/2+1 = 951 
G. Supermajority Requirements 
▪ Supermajority requirements give a minority shareholder veto power over corporate decisions without 
offending corporate norms because corporation statutes allow high quorum and voting requirements  
a. For Shareholders 
● Formalities for ​adoption  
o MBCA  
▪ Must be specified in the charter 
▪ Requires the same quorum or voting requirement as the proposed amendment 
o DGCL 
▪ Bylaws ok 
▪ Simple majority ok  
● Repeal or amendment 
o MBCA = Supermajority vote is needed 
o DGCL = supermajority only for Charter amendment; Bylaws simple majority ok 
b. For Directors 
● Formalities of Adoption  
o Charter or Bylaws 
o In close corporations, could be in a shareholder agreement 
H. Preemptive Rights   
▪ Default Rule: ​only have preemptive rights for the class of stock you own 
● Can dilute a person’s shares by creating a new class of stock because don’t have rights for that 
new class 
▪ No preemptive rights for employees because seen as compensation  
▪ Preemptive rights are the rights of a shareholder to subscribe to the portion of any increase in a 
corporation's capital stock necessary to maintain the shareholder's relative voting power as against other 
shareholders 
▪ Preemptive rights no longer are considered inherent in capital stock, but they may be granted or denied 
by the articles of incorporation 
▪ Both the DGCL and Model Act provide for an ​opt in to preemptive rights 
● The default rule is against preemptive rights  
● But a corporation may provide for preemptive rights by including a provision to that effect in 
the articles of incorporation  
▪ Most state corporation statutes include an opt out provision for preemptive rights 
43
▪Because preemptive rights complicate the issuance of shares, public companies rarely have such 
provisions 
▪ Preemptive rights are very common in closely held corporations 
I. Deadlock 
▪ Ways to resolve: 
i. Voluntary dissolution (approval of directors and shareholders; filing articles of dissolution) 
1. three step procedure just like an amendment to the charter 
ii. Judicial dissolution (less common remedy because very costly) 
1. dissolve the entity and wind-up the affairs 
2. ceases to exist - settle all accounts and payout 
iii. Mediation or arbitration (should include in organizational documents; include in bylaws in case 
of deadlock one of these is way to go – would save money) 
iv. Deadlock-breaking shareholder or director (can agree in advance who would be the 
deadlock-breaking director so if there ever is a deadlock, this person is temporarily a director to 
resolve the deadlock) 
1. good planner 
2. if you don’t have this, you are inviting litigation 
v. Buyout (one person wants to continue and the other disagrees, then that person could have their 
shares bought by the corporation in a buy-sell agreement) 
▪ Shawe v. Elting 
● quintessetntial example of deadlock situation 
● Shawe (49), Elting (former fiance) (50), and Shawe’s mother (1 - just for tax purposes) 
1. were engaged when they formed the corp 
2. lived in dorm together at NYU 
3. broke up and Shawe goes nuts - doesn’t take the break-up well and terrorizes the 
ex-fiance. Crawled under her bed twice and refused to leave 
J. Oppression of Minority Shareholder 
a. Basics 
● Under default rules, the majority will always control  
● Just because don't have a voice, does not automatically mean have a cause of action  
● Must sign a shareholder agreement to guarantee have voice when a minority shareholder 
b. For Suit of Oppression 
● Lack of market to sell the shares (no way to get out, investment locked in) 
o Applies in closely held corporations because generally there is no market for the shares 
(Wall Street Rule = if don't like the company, sell your shares) 
● Lack of management rights (contractual or otherwise) 
o Often they had some before and now they have been taken away = locked out 
● Must show unfairness in treatment (fired from board, no dividends being paid, no access to 
investment) 
● Leslie v. Boston Software Collaborative 
c. Responses to Shareholder Oppression 
i. Massachusetts:  
o majority owes fiduciary duty of “​utmost good faith and loyalty​.”   
o The test is whether controlling group can demonstrate a legitimate business purpose for 
its action. (the action is the unfairness; ex. – failure to pay dividend; courts deferential to 
business purposes) 
o Minority can then show that the same legitimate objective could have been ​achieved 
through an alternative course of action less harmful ​to the minority.  
44
o Heightened fiduciary duty of utmost good faith and loyalty. ​Donahue v. Rodd Electrotype 
Co.​   
o But no “straight jacket.” ​Leslie v. Boston Software Collaborative.  
o Controlling group must demonstrate a legitimate business purpose. Minority may then 
show that the same objective can be achieved through alternative means. ​Wilkes v. 
Springside Nursing Home.​    
o Courts have broad powers to determine appropriate relief. ​Leslie 
ii. Delaware:  
o no special judicially created rules to protect minority shareholders. (Most unfortunate 
state from perspective of minority shareholders) 
o Case where court said does not exist in DE = Nixon v. Blackwell  
iii. MBCA 
1. Minority View (adopted by majority of states) 
▪ Asks whether majority shareholders have acted reasonably from the minority’s 
point of view 
▪ And whether minority’s reasonable expectations have been met 
▪ The minority looks at the situation from the eyes of the minorities viewpoint 
● How does the minority view the situation 
● Look at the expectations of the minority  
2. Majority View  
▪ Reasonableness form point of view of majority  
▪ The majority view focuses on the justifications given by the majority 
(shareholders/directors because often the same because closely held corp.) 
▪ Here focusing on the legitimate business purpose given 
 
   

45
Duty of Care 
A. Standard of Conduct vs. Standard of Liability ​(the two don’t match) 
a. Standard of Conduct 
● MBCA 8.30 
1. (a) DUTY OF LOYALTY: ​each member of the board of direction when discharging 
the duties of a director, shall act: 
a. in good faith 
b. in a manner that the director reasonably believes to be in the best interest of the 
corporation 
2. (b) DUTY OF CARE:​The members of the board of directors . . . , when becoming 
informed in connection with their decision-making function or devoting attention to 
their oversight function, shall discharge their duties with the care that a person in a like 
position would reasonably believe appropriate under similar circumstances. 
a. more about how you do things 
3. c) . . . a director, who does not have knowledge that makes reliance unwarranted, is 
entitled to rely on the performance by any of the persons specified in subsection[s] (e)(1) 
or . . . (e)(3) . . . . 
4. (d) . . . a director, who does not have knowledge that makes reliance unwarranted, is 
entitled to rely on information, opinions, reports or statements, including financial 
statements and other financial data, prepared or presented by any of the persons 
specified in subsection (e). 
5. (e) A director is entitled to rely . . . , on:  
a. . . . o​fficers or employees ​of the corporation whom the director reasonably 
believes to be reliable and competent . . . or the information, opinions, reports or 
statements provided;  
b. legal counsel, public accountants,​ or other persons retained by the 
corporation as to matters involving skills or expertise the director reasonably 
believes are matters (i) within the particular person's professional or expert 
competence or (ii) as to which the particular person merits confidence; or  
c. a committee . . . ​of which the director is not a member if the director 
reasonably believes the committee merits confidence. 
b. Standard of Liability  
● MBCA 8.31 (this statute is not very helpful and is drawn from common law so case law is more 
helpful; in DE, there is not a statute so it is all common law) 
B. Misfeasance and the Business Judgement Rule (BJR) 
▪ Misfeasance = mismanagement 
▪ Business Judgement Rule 
● Applies in all situations unless involves fraud, illegality, or conflict of interest 
● BJR​ ​is a presumption that the board acts in good faith and in the best interest of the 
company 
● Courts​ ​generally are very favorable to actions taking by large corporations 
i. Application: 
o Schlensky v. Wrigley 
▪ We do not mean to say we have decided that the decision of the directors was a 
correct one. That is beyond our jurisdiction and ability. We are merely saying 
that the decision is properly one before directors and the motives alleged in the 
amended complaint showed no fraud, illegality or conflict of interest in their 
making of that decision 
46
o Cases arising under the Model Act have traditionally been analyzed in a manner similar 
to the Delaware cases 
o Business Judgement Rule - more like the rule of abstention. Substance of decision stands 
because the decision was not tainted by fraud, illegality, or conflict of interest  
i. presumption the board acted in good faith on behalf of the corporation 
ii. Justifications: 
o BJR avoids judicial interference in areas where judges have no expertise 
o Encourages directors to serve by limiting their exposure to liability 
o Matter of policy: permitting shareholders to enforce liability against directors for 
decisions that turn out badly is not a good idea (want to encourage risk taking) 
iii. Gagliardi v. Trifoods International, Inc. 
o founder of tri-foods (steak-ums) and still has 13% stock in the corporation after he left 
the company.  
a. disagreed with purchasing another manufacturing plant 
b. foolish to purchase a new product facility in PA 
c. gross negligence in sales volume 
o Where a director is independent and disinterested, there can be no liability for corporate 
loss, unless the facts are such that no person could possibly authorize such a transaction 
if he or she were attempting in good faith to meet their duty 
o To make out a viable claim of breach of fiduciary duty, plaintiffs must show self-dealing 
or improper motive 
a. disagreements for whether a decision was ​Foolishly risky! Egregiously risky! i​ s not 
within the court’s discretion (policy reasons) 
b. things not protected by the business judgment rule (what you would need 
to plead in order to survive a summary judgment motion) 
i. conflict of interest: ​one of the directors on both sides of the transaction 
(tainted transaction)  
ii. fraud: ​process deficiencies 
iii. illegality: ​violating some sort of duty imposed by law 
C. The Decision-Making Process 
▪ The circumstances under which courts become suspicious of board decisions fall into two categories 
● When directors are subject to a conflict of interest with respect to the challenged decision, 
courts review the decision under the standards developed under the duty of loyalty 
● When directors fail to gather the requisite information to make the challenged decision, courts 
review the decision under the gross-negligence standard 
▪ How would you figure out the value of a company 
● Intrinsic value is the perceived or calculated value of an asset, an investment, or a company. 
The term finds use in fundamental analysis to estimate the value of a company and its cash 
flows. Another use of intrinsic value is in the amount of profit that exists in an options 
contract. 
▪ Smith v. Van Gorkom (en banc - all judges hear the case instead of just a panel of 3) 
● VERY IMPORTANT DECISION and MUST remember by name 
● Talking about the decision making process because can't prevail on the substance but can prevail 
on the form -- trying to prevail against BJR 
● Pure negligence is never enough so the standard is gross negligence (something close to 
recklessness) 
● “We hold that the directors of Trans Union breached their fiduciary duty to their stockholders 
(1) by their failure to inform themselves of all information reasonably available to them and 
47
relevant to their decision to recommend the Pritzker merger; and (2) by their failure to disclose 
all material information such as a reasonable stockholder would consider important in deciding 
whether to approve the Pritzker offer.  
1. need to disclose all facts germaine to the transaction 
2. all material facts (objective standard) → substantial likelihood. 
● We hold, therefore, that the Trial Court committed reversible error in applying the business 
judgment rule in favor of the director defendants in this case.” 
● Deficiencies in decision-making process​ (Court looked at these factors)(one issue was the 
board meeting only lasted 2 hours) (Van Gorkom only made a presentation for 20 minutes)(did 
not review any documents) (Anytime see mention of the process on an exam and being asked to 
evaluate breach of fiduciary duties look to length of negotiation, length of meeting, amount of 
work put in before giving materials to board, how came up with the price, was it time pressured 
basically any indicators of timing and the amount of work that went into making the 
decision)​(theory for breach of duty of care -- it allows you to avoid the protection of the 
BJR) ​(another fact pattern may be getting an advice on accounting from someone who 
specializes in software) 
o Directors’ failure to inform themselves (This is the most important) 
o Price  
o Negotiations (flawed because no one else participated and over drinks between Pritzker 
and Van Gorkom) 
o Time pressures 
o Usurping the board’s functions (Board had to adopt resolution and then the 
shareholders must vote, Van Gorkom can't take over this function, it is non-delegable) 
o Failure to disclose to shareholders “all facts germane to the transaction at issue” (this 
was flawed because didn't include all material facts but since it wasn't probably 
investigated so couldn't disclose all the material facts) 
▪ DGCL 141(e) 
● A member of the board of directors, or a member of any committee designated by the board of 
directors, shall, in the performance of such member's duties, ​be fully protected in relying in 
good faith upon ​the records of the corporation and upon such information, opinions, ​reports 
or statements ​presented to the corporation ​by any of the corporation's officers or 
employees​, or committees of the board of directors, or by any other person as to matters the 
member reasonably believes are within such other person's professional or expert competence 
and who has been selected with reasonable care by or on behalf of the corporation. 
o Here no investigation was done so can't really rely on what was said  
▪ Legacy of Smith v. Van Gorkom (​ These are the ways corporate law was changed after this decision) 
● D&O liability insurance (won't cover intentional misconduct, covers conduct akin to Smit v Van 
Gorkom) 
● Director exculpatory statutes (must check articles of incorporation to see what company 
adopted; must be included in corporate charter) 
● Director and officer indemnification  
● Changes in decision-making process 
o “Investment Banker's Relief Act of 1985” (now investment bankers must be hired as 
experts so they make a lot more money; a court opinion not an act but this is what it is 
called) 
D. Nonfeasance and “Egregious” Conduct/The Waste Standard 
▪ Nonfeasance = no action taken at all  
▪ Francis v. United Jersey Bank 
48
● A director will be held liable if they don’t do anything at all  
▪ The Waste Standard (no one has ever prevailed under this standard) (This is the hypothetical one) 
● Applies in situations where the decision is so “egregious” that directors should be held liable for 
breach of duty of care 
o Even if the decision is not motivated by self-interest 
o Even if there is no evidence that managers failed to gather information 
● Not enough to impose liability 
● Court likely to still just apply BJR 
● Walt Disney case 
● Sheer size of payout seems so unreasonable, we are going to look at the actions of board of 
directors to decide if it is reasonable 
● survived motion to dismiss 
● court clarified MUST PLEAD WITH PARTICULARITY, facts that would amount 
to egregious conduct for waste 
● a sentence or two on exam if leaves a bad taste in mouth 
E. The Shareholder Primacy Norm 
▪ The true owners are the shareholders and every decision must be made in the best interest of the 
corporation and the shareholders 
▪ To whom do directors owe their duty of care 
● The Model Act requires directors to make decisions in the interests of the corporation and many 
cases state that the duty is owed to the corporation and its shareholders 
● Courts have often concluded that the shareholders are the primary beneficiaries of the duty of 
care 
o This aspect of the duty of care is often called the shareholder primacy norm 
▪ Dodge v. Ford Motor Co 
● The most commonly quoted judicial statement of the shareholder primacy norm was set out by 
the Michigan Supreme Court  
i. one of the clearest holdings 
● A business corporation is organized and carried on primarily for the profit of the stockholders. 
The powers of the directors are to be employed for that end. The discretion of directors is to be 
exercised in the choice of means to attain that end, and does not extend to a change in the end 
itself, to the reduction of profits, or to the non-distribution of profits among stockholders to 
devote them to other purposes 
▪ Kahn v. Sullivan ​(Modern Approach) 
● Have to tie every decision you make to the benefit of the stockholders 
● The Hammer museum was ok because it was good PR for the shareholders - increase people’s 
awareness which would make them buy more 
▪ Ebay v. Newmark 
● both companies (EBay and Craigslist) founded in 1995 
● hinges very much on the specifics of the contract, and how it was structured to create this sort of 
duty on Craigslist's part to eBay 
i. have a restriction on resale associated with the shares - decide to sell to ebay 
ii. 32 million dollars in total for minority stake in Craigslist 
● Stock purchase agreement 
i. reincorporation of business in Delaware 
ii. bargain for cumulative voting 
1. a system of voting in an election in which each voter is allowed as many votes as 
there are candidates and may give all to one candidate or varying numbers to several. 
49
a. ensures representation on the board. Say there are 3 directors and Ebay has 
28 shares. Multiply 28 X 3 and then they use all those votes for one board 
member seat. Would not have enough to pick all three board slots, but they 
could accumulate the votes for one 
iii. veto power (supermajority requirement) 
1. amendment to the charter 
2. influencing dividend declarations 
iv. Voting agreement 
1. Between Jim and Craig (did not have anything to do with EBay) 
a. neither one has majority stake individually but together they have over 70%.  
b. enter into vote POOLING agreement where they vote as a block - vote for 
each other’s nominees on the board. Ensure they can elect two reps on the 
board (it will be themselves) 
v. The specific number of shares are listed in the contract at the stated price. This agreement 
proves that the sale and the terms of it were agreed upon mutually. 
● Ebay was upset with investment (protect corporate culture) 
i. brought suit for restriction of rights as a stockholder (adopting defensive tactic) 
1. in-hands business judgment rule 
2. place burden on directors to identify some proper corporate objectives and justify 
actions as reasonable  
a. goal seeking to accomplish and the way I chose to accomplish is reasonable 
ii. Court held that Craigslist violated fiduciary duties and not in good faith 
1. Take away - The court in the case - corporate culture standing alone is worthy 
of protection as an end in itself. Tie corporate culture to stockholder value. All 
have to focus on protecting shareholder value.  
a. were a for-profit company - choose to take 32 million from outside investors.  
b. that investor has an expectation for the business. those are the new standards 
the company must abide by.  
● DOES THIS MEAN that literally malfeasance for a corporation not to do everything it legally can 
to maximize its profits??!?!? 
F. Fraud or Illegality 
a. Chiquita Bananas  
● They broke the law by authorizing the payments to terrorist’s organizations  
● If plead these facts, then the BJR will not apply but that doesn’t automatically mean you have 
won 
b. entire fairness test (how to look at duties that could have possibly been breached) 
● procedural and substantive fairness  
G. Exculpation  
a. MBCA 8.30​ = formulation for the duty of care for directors 
b. MBCA 2.2 02 ​= Exculpation (essentially same as DE) 
● The articles of incorporation may set forth:  
o a provision eliminating or limiting the liability of a director to the corporation or its 
shareholders for money damages for any action taken, or any failure to take any action, 
as a director, except liability for (These 4 things are the exceptions) 
▪ the amount of a ​financial benefit ​received by a director to which he is ​not 
entitled(stealing money or breach of duty)​;  
▪ an ​intentional infliction of harm ​on the corporation or the shareholders;  
▪ a violation of section 8.33 [​unlawful distributions​]; or  
50
▪ an ​intentional violation of criminal law 
c. Delaware 102(b)​ = Exculpation (first law responding to Van Gorkom) (Exculpatory clause of Raincoat 
provision) 
● The certificate of incorporation may also contain any or all of the following matters: 
o A provision eliminating or limiting the personal liability of a director to the corporation 
or its stockholders for monetary damages for breach of fiduciary duty as a director, 
provided such provision shall not eliminate or limit the liability of a director: (Then next 
4 things are exceptions) 
▪ For any ​breach of the director's duty of loyalty​ to the corporation or its 
stockholders; (this is a broader exception) 
▪ for ​acts or omissions not in good faith ​or which involve ​intentional 
misconduct or a knowing violation of law​;  
▪ under § 174 of this title [​unlawful distributions​]; or  
▪ for any transaction from which the director derived an ​improper personal 
benefit​. 
 
H. Analysis for case against Directors ​(This is all need to know) (​Duty of Care​ is about the process and how 
decisions are made) 
1) Courts will not second-guess the substance of decisions of directors (BJR) ​(always start here with 
BJR; say BJR is a presumption that board acted in good faith and for the benefit of the corporations. 
Courts will not second guess the director's business decisions because not as well equipped to as the 
directors to make the business decisions) (Then look for exceptions) 
a) Unless the decision is tainted with fraud or illegality; or ​(if can show the conduct or 
decision was in violation of the law of otherwise fraudulent, then BJR will not apply which 
means can move forward and try the case to the judge or jury -- doesn't mean you win) 
b) Unless the process that generated those decisions is unsound:  
i. When directors are subject to a conflict of interest with respect to the challenged 
decision, courts will review the decision under the duty of loyalty ​(1 deficiency in 
the process where BJR won’t apply) 
ii. When directors fail to gather the requisite information to make the challenged 
decision, courts review the decision under the “gross negligence” standard; or 
(this is Van Gorkom; review under gross negligence standard -- can't be just forgot to do 
something) 
▪ Approval by majority vote of shareholders cures failure to reach informed 
decision 
c) Unless the director utterly fails to perform his/her duties; or​ (claim for nonfeasance -- don't 
do anything at all) ​Francis v. United Jersey Bank 
d) Unless the directors’ decision was substantively “egregious” (analyzed under the waste 
standard). ​(some conduct is so horrible that the BJR should not protect it)  
 
   

51
Duty of Loyalty 
A. Duty of Loyalty and Conflict-of-Interests Transactions 
▪ MCBA 8.30 = Standard of conduct for Directors (formulation for duty of loyalty) 
● Each member of the board of directors, when discharging the duties of a director, shall act: 
o In ​good faith​, and 
o In a ​manner the director reasonably believes to be in the best interests of the 
corporation​. (this doesn't say much because statute very skeletal so need common law) 
Reasonable person would act.  
▪ Corporation: Duty of Loyalty in General​ ​(Duty of Loyalty violations can't be exculpated and BJR 
doesn't apply to breach of Duty of Loyalty) 
● Director (or Officer) stands of​ ​both sides of the transaction​ ​or otherwise receives a​ ​personal 
benefit. ​Once the plaintiff has established that, the burden shifts to the defendant. Utensil  
● Classic fact pattern:  
o Engaging in a competing venture 
o Self-dealing 
o Usurpation of corporate opportunity 
a. directors, officers, and executives of companies may not exploit for their 
personal gain an idea presented to them or developed by them in their corporate 
capacities 
i. think Meinhardt v. Salmon - renews lease for himself only. Usurping 
business opportunities.  
ii. disclose all facts to corporation, allow them to decide whether or not to 
pursue the transaction 
● Conflict of Interests: BJR does not apply 
● Burden is placed on the fiduciary (if Director A works for coffee shop and is buyer and seller in 
the transaction -- this is all you need to plead, then the burden is placed on Director A and they 
must meet the entire fairness test) (for burden to shift the plaintiff must plead a conflict of 
interest) 
● Fiduciary must act with complete candor 
o disclose to board and shareholders  
o full disclosureof all material facts 
● Court will apply the ​“entire fairness” test​ (focus on substantive (fair price) and procedural (fair 
dealing) fairness) 
o study? Appraisal? What was done?  
o Burden starts with fiduciary 
i. Competing with the Corporation 
● Director wants to engage in side business that competes directly with Company A 
● Typical scenarios: 
o Fiduciary leaves Company A to join Company B (misappropriation, unfair competition, 
breach of a non-compete); 
o Poaching cases (recruitment of personnel and clients)  
● Jones Co., Inc. v. Burke-​Duane Jones won a jury verdict, and the judgment was affirmed on appeal: 
defendants breached the duty of loyalty by setting up their new business while still employed 
with Duane Jones. They were soliciting other clients.   
ii. Self-Dealing Transactions (​Bayer v. Beran ​the directors wife used to sing in ad. Plaintiff lost because it 
was found to be to the benefit of the company and there was no evidence that there was a better singer 
out there) 

52
● Fiduciary is on ​both sides​ of the transaction (defined in MBCA § 8.60(1)): any transaction to 
which the fiduciary is a party or in which she has a known “​material financial interest​,” or in 
which a “​related person​” is a party or has a material financial interest 
o A material financial interest is one that reasonably would be expected to impair one’s 
objectivity 
a. even if it is not a person related (just say, a friend) -- it would still be tainted 
because objectivity would be impaired. 
o A related person: fiduciary’s spouse or relative, a person living in fiduciary’s home, or an 
entity controlled by the fiduciary 
● Safe Harbor for Self-Dealing transactions: 
o Traditional Rule​: the tainted transaction is void. (now say transactions are voidable 
because have safe harbor provisions that apply)​ (Today transactions are voidable 
unless it is entirely fair) 
▪ Courts gradually moved from void standard to voidable standard, recognizing 
self-dealing transactions can be beneficial to corporation 
o State have now adopted interested director statutes  
▪ In self-dealing transactions, the burden is on the defendant fiduciary to show that 
the transaction was fair if defendant meets burden then they are protected by the 
BJR.  
▪ Fairness is assessed through the “intrinsic fairness” (“entire fairness”) test: 
● Process of approval (“fair dealing”) and (procedural fairness), Enough 
time was spent, experts hired, due diligence into the decision.  
● The terms must be fair to the corporation (“fair terms”). ​Weinberger v. 
UOP, Inc.,​ 457 A.2d 701 (Del. 1983). (substantive fairness) 
o Holding of Weinberger, the burden is on the fiduciary to show 
fairness 
● MBCA 8.61: 
o Interested director transactions are not voidable because of conflict of interest if 
fiduciary shows​: 
▪ That the transaction was approved by disinterested shareholders,​ ​or 
▪ That the transaction was approved by disinterested directors,​ ​or 
▪ That the transaction was fair to the corporation when entered 
o Strong form statute: judicial scrutiny of fairness is limited​ ​if there is either disinterested 
board or shareholder approval 
● DGCL §144 
o No​ contract or transaction between a corporation and 1 or more of its directors or 
officers​, or between a corporation and any other corporation, partnership, association, 
or other organization in which 1 or more of its directors or officers, are directors or 
officers, or have a financial interest, ​shall be void or voidable solely for this reason​, or 
solely because the director or officer is present at or participates in the meeting of the 
board or committee which authorizes the contract or transaction, or solely because any 
such director's or officer's votes are counted for such purpose, ​if: 
▪ The ​material facts ​as to the director's or officer's relationship or interest and as 
to the contract or transaction ​are disclosed or are known ​to the board of 
directors or the committee, and the board or committee in good faith authorizes 
the contract or transaction by the ​affirmative votes of a majority of the 
disinterested directors​, even though the disinterested directors be less than a 
quorum; ​or 
53
▪ The ​material facts ​as to the director's or officer's relationship or interest and as 
to the contract or transaction ​are disclosed or are known ​to the stockholders 
entitled to vote thereon, and the contract or transaction is ​specifically approved 
in good faith by vote of the stockholders​; ​or 
▪ The contract or transaction is ​fair as to the corporation ​as of the time it is 
authorized, approved or ratified, by the board of directors, a committee or the 
stockholders. ​(Must still disclose all material facts, otherwise the taint is not 
removed) 
o Common or interested directors may be counted in determining the presence of a 
quorum at a meeting of the board of directors or of a committee which authorizes the 
contract or transaction. ​(Can count the tainted directors in determining the 
quorum) 
● Kahn v. Lynch Comm’n Systems 
o The approval by disinterested directors or shareholders has the effect of​ shifting the 
burden of proof of unfairness to the plaintiffs 
▪ If the self-dealing transaction was approved by disinterested directors or 
shareholders, that approval removes the taint of conflict of interest. 
▪ The burden is on the plaintiff to show the deal was nonetheless unfair to the 
corporation. 
▪ Fairness in this case is assessed under BJR. 
o Compare to MBCA approach (approval of directors or shareholders ends the court’s 
inquiry). 
● Duty to Disclose 
o The material facts must always be fully disclosed to the board or shareholders 
● HMG/Courtland Properties, Inc. v. Gray  
● Be Benihana of Tokyo v. Benihana  
iii. Analysis: 
● when have someone on both sides of the transaction: (Conflict of Interest Transaction) 
o Because of the presence of a conflict of interest the burden is on the defendants 
to show the transaction was fair to the corporation 
▪ Show this fairness by either demonstrating (If the defendant proves one of 
the safe harbors removed the taint from the transaction, then the burden 
shifts back to the Plaintiff who then must overcome the BJR)​ (In ​MBCA 
which is minority view, once show a safe harbor there is not more burden 
shifting) 
● Disinterested shareholders approved (MUST disclose all material 
facts to remove the taint -- Gray) (Need majority approval) = safe 
harbor 
o If the self-dealing transaction was approved by disinterested 
directors or shareholders, that approval removes the taint of 
conflict of interest. 
o The burden is on the plaintiff to show the deal was nonetheless 
unfair to the corporation. 
o Fairness in this case is assessed under BJR 
● Disinterested directors approved (MUST disclose all material facts 
to remove the taint -- Gray) (Need majority approval) = safe harbor 

54
o If the self-dealing transaction was approved by disinterested 
directors or shareholders, that approval removes the taint of 
conflict of interest. 
o The burden is on the plaintiff to show the deal was nonetheless 
unfair to the corporation. 
o Fairness in this case is assessed under BJR 
● Entire fairness test was met 
o Fairness is assessed through the “intrinsic fairness” (“entire 
fairness”) test:  
▪ process of approval (“fair dealing”) and (procedural 
fairness) 
▪ the terms must be fair to the corporation (“fair 
terms”). ​Weinberger v. UOP, Inc.,​ 457 A.2d 701 (Del. 
1983). (substantive fairness) 
● Holding of Weinberger, the burden is on the 
fiduciary to show fairness  
iv. Usurpation of Corporate Opportunity 
● Taking advantage of something that should belong to the corporation. If yes, then the 
corporation should be given the chance to invest in the opportunity.  
● If investment opportunity seen as belonging to the corporation, the corporation should be given 
a chance to invest in it.   
● Some courts require a fiduciary to present the opportunity to the corporation (right of first 
refusal). 
● Corporation may relinquish the opportunity in favor of the fiduciary 
o The relinquishment is an “interested director transaction,” subject to rules on 
self-dealing 
● No liability if corporation is incapable of pursuing the opportunity: 
o Third party may refuse to deal with the corporation. 
o Corporation may not be able to afford it financially 
▪ Some courts disallow fiduciaries to assert this defense 
● Statutory Formulation 
o Each member of the board of directors, when discharging the duties of a director, shall 
act in good faith, and in a manner the director reasonably believes to be in the best 
interests of the corporation. MBCA 8.30. 
● Delaware test for deciding if corporate opportunity 
o Broz v. Cellular Information Systems, Inc. ​(not assigned) ​use the four factor test from GUTH. 
All factors need to be addressed. This is an “equitable doctrine” so keep in mind, totality 
of the Circumstances as “what is fair…) 
● Broz is △; sole owner of corporation operating in midwest and director in 
another corporation that is not in Midwest 
▪ both companies operating cell phone networks 
● third party issues licenses for wireless networks (FCC - federal communication 
commission) 
o Guth v. Loft 
▪ Corporate officer or director ​may not take​ a business opportunity for his own 
if: (this is the framework for analyzing usurpation cases) 
● the​ ​corporation is ​financially able to exploit ​the opportunity;  
● the opportunity is within the corporation's ​line of business​;  
55
● what is the business that is being operated (both selling kid’s 
apparel?) 
● the corporation has an ​interest or expectancy ​in the opportunity; and  
● employees to develop side business 
● use hours, resources, money  
● by taking the opportunity for his own, the corporate fiduciary will 
thereby be placed in a ​position inimical ​(hostile)​ to his duties to the 
corporation 
o Corporate officer ​may take ​a business opportunity for his own if:  
● the opportunity is presented to the director or officer in ​his 
individual and not his corporate capacity 
● the opportunity is not ​essential to the corporation 
● the corporation holds ​no interest or expectancy ​in the 
opportunity AND 
● the director or officer has not ​wrongfully employed the 
resources ​of the corp is pursuing or exploiting the opportunity 
● presenting the opportunity to the corporation serves as a safe harbor but failure to 
present it does not automatically result in liability - merely shifting burden  
● American Law Institute Principle of Corporate Governance test ​(Secondary law source; 
must present the opportunity) Some jurisdictions choose to adopt this test.  
o Northeast Harbor Golf Club v. Harris (Mrs. Harris is part owner of golf course; shareholder and 
owner of golf course; purchases extra land for herself and starts running a competing business; Golf club 
files suit for breach of fiduciary duty) 
▪ absolute requirement​ that the fiduciary present the opportunity to the 
corporation ​and 
i. wait for it to reject it 
1. Rejection must be fair to the corporation, ​or  
2. the decision to reject the opportunity must be by disinterested 
directors or shareholders 
o Definition: Corporate opportunity is: 
▪ either something the fiduciary becomes aware of in connection with the 
performance of her corporate function or  
▪ through use of corporate information or property, or  
▪ something the fiduciary is aware of that is “closely related to a business in which 
the corporation is engaged or expects to engage.” 
i. very similar to Delaware Law 
● Financial inability to pay is not a defense 
 
● Remedy for Conflict-of-Interest transactions 
1. Rescission of the deal 
2. Constructive trust on property 
3. Corporation must be put in the position it should have been in absent breach 
B. Controlling Shareholders’ Duties 
a. Sinclair Oil v. Levien 
● Controlling shareholders owe minority shareholders fiduciary duties 
● The court applied 2 standards to assess the fairness of the dealings between parent and 
subsidiary 
o BJR 
56
o Entire Fairness  
● If ​have self-dealing transaction involving a controlling shareholder: 
o If the minority shareholder is being treated equally: 
▪ The ​burden will shift to defendant to prove fairness 
● If ​they prove a safe harbor, the burden will shift back to the Plaintiff to 
overcome the BJR 
o If the minority shareholder is not being treated equally 
▪ Then when the burden shifts back to the plaintiff they apply the more stringent 
(heightened) standard of entire fairness 
b. Williamson v. Cox Communications, Inc.  
● A ​shareholder doesn’t owe fiduciary duty​ to other shareholders unless she a ​controlling 
shareholder 
o The test for control: 
▪ Owning more than 50% of the voting power or 
▪ Exercising control over the business and affairs of the corporation 
o Must show actual control; potential ability to exercise control is not enough 
o No need to show actual control over day-to-day operations; actual control with regard to 
the challenged transaction is enough 
o Where a controlling shareholder is on both sides of the transaction, the transaction is 
viewed under the ​entire fairness standard 

 
▪ A shareholders’ approval doesn't automatically extinguish the breach of duty of loyalty claims. 
Ratification should be viewed together with other factors. Ratification imposes a burden of proof on 
plaintiffs using the BJR standard 
C. Officer Fiduciary Duties 
▪ Gantler v. Stephens 
● “In the past, we have implied that officers of Delaware corporations, like directors, owe 
fiduciary duties of care and loyalty, and that the fiduciary duties of officers are the same as those 
of directors. We now explicitly so hold.” 
● Officers are agents of the board so not apparent that they owe fiduciary duties 
D. Controlling Shareholder Fiduciary Duties 
▪ Williamson​ = Fiduciary Duties of Controlling Shareholders 
● Approval of a controlling shareholder transaction by the majority of the minority of 
shareholders shifts the burden to the plaintiff. Standard of review is entire fairness. ​In re 
Wheelabrator 
57
E. Oversight and Good Faith ​What it means to act in good faith after the exculpation statutes were adopted 
a. In Re Caremark 
● “Only a ​sustained​ or ​systematic​ failure of the board to exercise oversight—such as an utter 
failure to attempt to assure a reasonable information and reporting system exists—will establish 
the lack of good faith that is a necessary condition to liability.” 
o The director's responsibility is to ensure have mechanisms for director's to learn about 
violations 
o The test of liability is lack of good faith as evidenced by sustained or systematic failure to 
exercise reasonable oversight is quite high 
a. level of detail appropriate for the system is a “QUESTION OF BUSINESS 
JUDGMENT” 
o unconsidered inaction 
● Caremark Claim: (very similar to nonfeasance) (in this claim trying to avoid the applicability of 
an exculpatory clause so must show not in good faith) (how do you prevail? Prove omission not 
in good faith = Utter failure to oversee corporation by the directors) 
o Directors breached their duty of care by failing to adequately control the company’s 
employees. 
o Plaintiffs must show: 
▪ that directors knew or should have known that the violations of law were 
occurring, and 
▪ should have adequately informed themselves of the care of corporation 
▪ that directors took no steps in a good faith effort to prevent or remedy the 
situation, and 
▪ that such failure proximately resulted in the losses complained of 
b. In re Walt Disney Company Derivative Litigation 
● Proving bad faith requires more than acting without information and inadequate deliberation 
which is a duty of care analysis. “A failure to act in good faith may be shown, for instance, 
where the fiduciary intentionally acts with a purpose other than that of advancing the 
best interests of the corporation,​ where the fiduciary acts with the intent to violate applicable 
positive law, or where the fiduciary intentionally fails to act in the face of a known duty to act, 
demonstrating a conscious disregard for his duties.” 
c. Stone v. Ritter 
● bank that had a ponzi scheme occurring inside. Basically saying there was a great investment 
opportunity 
● Failure to act in good faith does not itself impose liability but does so because good faith is a 
“subsidiary element, i.e., a condition of the fundamental duty of loyalty.” 
● Good faith not an independent fiduciary duty, but the lack of good faith means that the 
directors are not acting in the best interest of the corporation and are thus breaching their duty 
of loyalty. 
Talk about what an exculpatory clause is, talk about exceptions, mention one possibility of bad faith, to 
establish a breach of duty of loyalty you need to meet the Walt Disney standard.  
F. Analysis 
▪ Traditional analysis​: Director stands of both sides of the transaction or otherwise receives a personal 
benefit. 
a. Competing with corporation 
i. Poaching cases. ​E.g., Jones Co., Inc. v. Burke​. 
b. Conflict-of-interest transactions 
i. Definition of interested director transactions. MBCA 8.60. 
58
ii. BJR does not apply; burden is placed on the fiduciary to show “entire fairness” of the 
transaction. ​Weinberger v. UOP, Inc.​  
iii. Unless a safe harbor applies: An interested director transaction is not voidable because 
of the conflict of interest if the fiduciary shows: 
1. That the transaction was approved by disinterested shareholders, or 
2. That the transaction was approved by disinterested directors, or 
3. That the transaction was fair to the corporation when entered. 
ii. Approval by disinterested directors or shareholders must occur after full disclosure of 
material facts to the board or shareholders. ​HMG v. Gray.​  
1. If the self-dealing transaction was approved by disinterested directors or 
shareholders, that approval removes the taint of conflict of interest. The burden 
is on the plaintiff to show the deal was nonetheless unfair to the corporation. 
Fairness in this case is assessed under BJR. ​Kahn V. Lynch Comm’n Systems.​  
a. Usurpation of corporate opportunity 
i. Test for corporate opportunity.   
1. Delaware approach. ​Guth v. Loft​; ​Broz v. Cellular Information Systems​. 
2. ALI Principles of Corp. Governance approach. ​Northeast Harbor Golf Club v. 
Harris 
 
Statutory Exculpation from Liability, Insurance, and Indemnification  
A. Exculpatory Clauses ​(MUST be included in your Charter) 
a. MBCA 8.30 ​= Standards of Conduct 
b. MBCA 8.31 ​= Standards of Liability 
c. MBCA 2.02 ​= Exculpation 
● The articles of incorporation may set forth: . . .  
o a provision eliminating or limiting the liability of a director to the corporation or its 
shareholders for money damages for any action taken, or any failure to take any action, 
as a director, except liability for  
i. the amount of a ​financial benefit ​received by a director to which he is ​not 
entitled​;  
ii. an ​intentional infliction of harm ​on the corporation or the shareholders;  
iii. a violation of section 8.33 [​unlawful distributions​]; or  
iv. an ​intentional violation of criminal law 
d. 8 Del. C. § 102 (b) ​= Exculpation  
● The certificate of incorporation may also contain any or all of the following matters: 
o A provision eliminating or limiting the personal liability of a director to the corporation 
or its stockholders for monetary damages for breach of fiduciary duty as a director, 
provided that such provision shall not eliminate or limit the liability of a director:  
▪ For any ​breach of the director's duty of loyalty​ to the corporation or its 
stockholders; 
▪ for ​acts or omissions not in good faith ​or which involve ​intentional 
misconduct or a knowing violation of law​;  
▪ under § 174 of this title [​unlawful distributions​]; or  
▪ for any transaction from which the director derived an ​improper personal 
benefit 
B. Insurance ​(very expensive especially after Van Gorkom) 
▪ D&O Insurance (language of the contract will govern) 

59
● Insurance coverage for corporate and securities liability, including management and defense of 
lawsuits when a lawsuit states a claim for an “insurable event.” 
o Insurable events are listed in the policy 
● No coverage for intentional misconduct, dishonesty, and breaches of duty of loyalty 
o This applies not matter what, no insurance will cover these 
▪ However, the language of the contract will always govern (the insurance policy) 
▪ Reimbursement insurance 
● Reimbursement to the company for any payments to third parties or the government (penalties, 
judgments, settlements, and the costs of defense) arising from actions of directors and 
employees. 
● Policy usually requires the insurance company to defend the insured in a lawsuit 
C. Indemnification 
▪ Typically companies will enter into indemnification agreements with officers and directors (usually rights 
are specified in bylaws). 
▪ Indemnification of directors and officers for any losses arising out of their service to the corporation. 
▪ Key issue: whether the directors, officers, employees, or agents (DOEA) acted in good faith and in a 
manner reasonably anticipated to be in corporation’s best interest. If DOEA are successful on the 
merits, the company ​must​ indemnify them for attorney’s fees and expenses. 
● In some instances, indemnification is permissive, but if successful on the merits Must indemnify 
because assumed to have acted in good faith  
▪ Indemnification: advancement of expenses 
● DGCL 145(e) and MBCA 8.53 both allow for advancement of expenses prior to final 
disposition of lawsuit. 
● Many corporations make advancement mandatory 
D. Forbearance Agreements 
▪ Forbearance (“standstill”) agreements 
● Freeze DOEA’s and company’s claims against the other for a specified period of time; provides 
other protections (such as statute of limitations waiver). ​E.g.,​ Problem 9-5. 
o Corporation may want to argue to third parties was no violation of law and yet seek 
compensation (maintain its right of contribution) against its DOEA if company is forced 
to pay money due to a judgment or a settlement, especially where DOEA acts were in 
violation of a company policy 
 
Shareholder Litigation 
A. Procedural Requirements 
1. Plaintiffs must have been shareholders at the time of the alleged breach of duty (“​contemporaneous 
ownership rule​”). Del. Ch. Ct. Rule 23.1. (can't buy into the lawsuit, must have been shareholders at 
the time of breach) 
a. otherwise you could just buy shares and then sue people.  
2. Plaintiffs must remain shareholders throughout the litigation (“​standing requirement​”). ​Lewis v. 
Anderson,​ 477 A.2d 1040 (Del. 1984). (if sell stock after start litigation, you have lost your standing and 
can no longer sue) 
3. Shareholders must demand that the board of directors take action before the shareholder assumes 
control of the litigation (“​demand requirement​”). Del. Ch. Ct. Rule 23.1.  
a. put together demand letter (see blow) 
4. Once a derivative claim is filed, the​ court must approve any settlement​. Del. Ch. Ct. Rule 23.1. 
a. just a lawyer acting so it should be fair to the 𝝥  
5. Also:​ adequacy of representation and bond requirement. 
60
● Must post the bond (the plaintiffs) and can't just use any attorney 
B. Demand Requirement 
a. MBCA Approach 
● § ​7.42. Demand. (universal demand requirement) 
o No shareholder may commence a derivative proceeding until: 
▪ a written demand has been made upon the corporation to take suitable action; 
and 
▪ 90 days have expired from date demand was made unless the shareholder has 
earlier been notified that the demand has been rejected by the corporation or 
unless irreparable injury to the corporation would result by waiting for the 
expiration of the 90-day period.  
o They will send a demand letter 
▪ In this letter must plead with particularity what exactly the demand is and what 
you thought was done wrong. The board needs to know what you are trying to 
achieve 
b. Delaware Approach 
● The Traditional approach 
● Required ​(here telling board to file against a third party) 
o If demand is made, and directors refuse to act, court will review board’s decision to 
refuse demand (“wrongful refusal” cases). 
o Decision to refuse the demand is reviewed under BJR. 
o Will never win a wrongful refusal suit because BJR applies 
● Excused​ (demand is futile if directors wouldn’t be willing to act upon demand as result conflict 
of interest or if challenged action presents a real risk of fiduciary liability)(board can’t be 
expected to sue itself) 
o More careful scrutiny if the allegations in the plaintiff’s complaint raise a ​reasonable 
inference ​that the BJR is not applicable for purposes of considering a pre-suit demand 
pursuant to Rule 23.1.   
o “[W]here officers and directors are under an influence which sterilizes their discretion, 
they cannot be considered proper persons to conduct litigation on behalf of the 
corporation. ​Aronson v. Lewis 
c. Demand Futility in Delaware 
● Aronson v. Lewis 
o In determining demand futility the Court of Chancery in the proper exercise of its discretion must decide 
whether, under the particularized facts alleged, a reasonable doubt is created that 
▪ The directors are disinterested and independent [or] 
▪ Challenged transaction was otherwise product of a valid exercise of business judgment 
● Beam ex rel. Martha Stewart Living Omnimedia v. Stewart 
d. Futility Analysis 
● Directors are entitled to a presumption that they were faithful to their fiduciary duties. The burden is on the 
plaintiff to overcome that presumption. 
● Court must determine whether a plaintiff has alleged particularized facts creating a reasonable doubt about a 
director’s independence to rebut that presumption. 
● Pleaded facts must create a reasonable doubt majority of board could have acted independently in responding to a 
demand 
e. Demand Futility Tests  
● In determining demand futility the Court of Chancery in the proper exercise of its discretion must decide whether, 
under the particularized facts alleged, a reasonable doubt is created that the directors are disinterested and 
61
independent or that the challenged transaction was otherwise the product of a valid exercise of business judgment. 
Aronson v. Lewis. 
● To satisfy the second prong, the plaintiffs must allege particularized facts sufficient to raise (1) a reason to doubt 
that the action was taken honestly and in good faith or (2) a reason to doubt that the board was adequately 
informed in making the decision. 
● Where the charter provides for director exculpation, plaintiffs must plead particularized facts that demonstrate 
that there was an i​ ntentional dereliction of duty or a conscious disregard for directors’ 
responsibilities, amounting to bad faith.​  
● When a plaintiff complains of board inaction, there is no challenged transaction. Plaintiff must plead 
particularized facts that create a r​ easonable doubt that, as of the time the complaint was filed, the 
board of directors could have properly exercised its independent and disinterested business 
judgment in responding to a demand.​ Rales v. Blasband. 
f. Director Independence 
● A director will be considered unable to act objectively with respect to a pre-suit demand if he or she is interested in 
the outcome of the litigation or is otherwise not independent. 
o A director’s interest may be shown by demonstrating a potential personal benefit or detriment to the 
director as a result of the decision. 
o The primary basis upon which a director’s independence must be measured is whether the director’s 
decision is based on the corporate merits of the subject before the board, rather than extraneous 
considerations or influences. 
● Lack of independence due to personal relationship 
o Plaintiff must plead particularized facts that create a reasonable doubt sufficient to rebut the presumption 
of director independence. 
o The relationship must be of a bias-producing nature: it must be so close that the director’s independence 
may reasonably be doubted 
o The doubt may arise either because of financial ties, familial affinity, a particularly close or intimate 
personal or business affinity or because of evidence that in the past the relationship caused the director to 
act non-independently 
C. Direct vs. Derivative Suits

 
 
▪ Direct example: Minority Oppression; preemptive rights 
▪ Eisenberg v. Flying Tiger Line, Inc. (​ famous 2nd Cir. Case; federal case that applies NY law) 
62
▪ Tooley v. Donaldson, Lufkin, & Jenrette, Inc. (​ This is the test that is applied) 
D. Special Litigation Committees (SLC) 
a. Zapata Corp. v. Maldonado 
● First, the court should inquire in the​ independence ​and​ good faith ​of the committee and the 
bases supporting its conclusions​. Limited discovery may be ordered to facilitate such 
inquiries. The corporation should have the burden of proving independence, good faith and a 
reasonable investigation, rather than presuming independence, good faith and reasonableness. 
● The second step provides the essential key in striking the balance between legitimate corporate 
claims as expressed in a derivative stockholder suit and a corporation’s best interests as 
expressed by an independent investigating committee. The​ court should determine, applying 
its own independent business judgment, whether the motion should be granted. ​(motion 
to dismiss) 
b. Test of SLC Independence (court makes own business judgement on the independence) 
i. The question of independence turns on whether a director is,​ for any substantial reason​, 
incapable of making a decision with only the best interests of the corporation in mind 
ii. Cases focus on​ impartiality ​and​ objectivity 
iii. A director may be compromised if he is beholden to an interested person. Beholden does not 
mean just owing in the financial sense, it can also flow out of personal or other relationships to 
the interested party 
iv. Independence is measured contextually, based on facts known to the court about the directors 
specifically (subjective actual-person standard) 
E. Shareholder Suits 
 

 
▪ How this works​: 
● First ask if direct or derivative suit, because if direct suit not subject to the same procedures as a 
derivative suit 
63
o To decide ask who is harmed? Who should get the recover? 
▪ Derivate = breach of fiduciary duty 
▪ Direct = to enforce preemptive rights, should be able to vote on something and 
wasn't able to 
o If have derivate must look at demand requirement 
▪ Is it required? 
● In MBCA, always required 
● A demand letter will go to the board and asking them to file suit 
▪ Excused? 
● When it is futile (Aronson v. Lewis) (conflict of interest) 
o Plead particular facts  
▪ Not disinterested shareholders, or  
▪ Reasonable doubt decision not subject to BJR 
▪ If excused, corporation will form an SLC  
● The SLC will decide what to do 
o Go ahead with suit or dismiss 
o Burden on SLC and board to prove independence of SLC 
▪ Zapata v. Maldonado 
▪ If required and made demand 
● If board doesn't pursue will be subject to BJR 
o By making the demand you accepted that the board was not 
tainted 
 
Closely held Corporation- 
A small business startup: 
● Who is going to finance the venture? 
● Who makes the management decisions? 
● Who is at risk if the venture loses money? 
● How do the founders exit the venture? 
● Taxation? 
● Flexibility 
● Choice of entity.  

64

Das könnte Ihnen auch gefallen